SlideShare una empresa de Scribd logo
1 de 210
Arrhythmias M. LaCombe MDFMR/UNECOM July 22, 2009
Let’s start with an easy one:
Sinus rhythm with 2:1 AV heart block. The ECG shows a bradyarrhythmia with non-conducted sinus P waves alternating with normally conducted P waves. It is not possible to reliably identify the point of block (nodal vs. infranodal) from this single ECG with 2:1 conduction. There is no evidence of acute inferior ischemia, either. The site of block could be proximal (in the AV node) or more distal, in the His-Purkinje system. In general, with 2:1 block, involvement of the AV node is favored by a narrow QRS complex and a prolonged PR interval, or by the presence of intermittent AV Wenckebach. Block (infranodal) in the His-Purkinje system would be favored by a concomitant bundle branch block and/or with a PR interval of 160 ms or less. A possibly useful bedside diagnostic test for chronic 2:1 block (in the absence of active ischemia) would be to increase the sinus rate (mild exercise). A resumption of 1:1 conduction favors AV node block while worsening of block strongly favors infranodal disease. Pacemaker placement is indicated for symptomatic 2:1 block without reversible cause (e.g., drug effect) and generally for asymptomatic 2:1 block due to infranodal disease. Intracardiac His bundle electrogram would definitively identify the site of block. This patient had intermittent 3:2 AV Wenckebach at other times, and then resumed 1:1 conduction on subsequent ECGs, consistent with AV node disease
Put this one together for me:
Sinus rhythm with AV Wenckebach with 4:3 conduction in the setting of an acute inferior wall infarction. The ECG demonstrates Q waves and ST elevation in leads 2, 3, and aVF. There are also reciprocal ST segment depressions in leads 1, aVL and V2-3. The rhythm is Wenckebach showing progressive prolongation of the PR intervals, shortening of the R-R intervals and block of every fourth P wave. The presence of "group" beating is easily recognized and characteristic of Wenckebach and is often associated with high vagal tone or nodal ischemia in the setting of an inferior wall myocardial infraction (MI). The block is at the level of the AV node
You’ve seen this one before, but now, let’s focus on the rhythm
Atrial fibrillation with a rapid ventricular response. Right axis deviation. Intraventricular conduction delay. Right ventricular hypertrophy (RVH). This patient has severe mitral stenosis with pulmonary congestion, pulmonary hypertension, and right heart failure. A qR in lead V1 is one of the most specific signs for right ventricular hypertrophy. Other causes of a qR in V1 include a right bundle branch block (RBBB) in patients with an anterior myocardial infarction or normal patients who have a QS in V1 and subsequently develop a RBBB. This patient has right axis deviation in addition to the qR in V1 which suggests RVH. Given atrial fibrillation and RVH one should strongly consider mitral stenosis. Additional note on differentiating right ventricular hypertrophy (RVH) from lateral MI: In some cases of severe (RVH) with marked right axis deviation, leads I and aVL may show rS or even QS waves, simulating lateral MI. In this case, lead V1 notably shows a narrow qR complex (not just a tall R wave) strongly suggestive of RVH, rather than posterior-lateral MI. Also, the lateral precordial leads here show rS type complexes, not QR waves. Physical examination, echocardiogram and other imaging modalities also provide important information about RVH vs. lateral MI in cases where the ECG is ambiguous.
 
The tracing shows sinus rhythm with complete heart block and an A-V junctional type escape rhythm. The P-P interval surrounding an individual QRS complex is narrower (shorter) than the P-P interval between two QRS complexes. Sinus rate variation with complete heart block is called ventriculophasic sinus arrhythmia. The patient has congenital complete heart block. This entity is usually idiopathic but may be secondary to placental transfer of anti-Ro and La antibodies from the mother. The spectrum of disease ranges from patients who are born with severe bradycardia and require pacemakers in infancy to patients who live full life spans without artificial pacemakers. The overall infant mortality is 15%. The escape rhythm in this instance is proximal in the conduction system and a pacemaker is not required at this time.
 
Ventricular paced rhythm with underlying complete heart block (P waves march through). Most important is superimposed current of injury with hyperacute ST-T changes inferiorly and laterally with reciprocal change V1-V3 consistent with acute infero-postero-lateral ischemia/ myocardial infarction (MI). Pacemaker patterns, like left bundle branch block (LBBB), often mask acute or chronic MI. But sometimes, as in the present case, the ischemic changes "show through."
 
Acute infero-lateral and probably posterior myocardial infarction (MI) with inferior Q waves and hyperacute ST-T complexes inferiorly and laterally with reciprocal ST depressions V1-V3. Tall initial R waves in V1-V3 with right bundle branch block. There is second degree AV block (Type 1) with 2:1 block initially and then 3:2 AV Wenckebach with underlying sinus rate of about 85/min. Left axis deviation is present. However, since this is an inferior MI, a pacemaker is not usually indicated: block here is in the AV node usually due to ischemia and increased vagal tone. In contrast, new right bundle with left axis with acute anterior-septal MI would be indication for prophylactic pacemaker. AV block in that setting is due to Type II mechanism associated with severe involvement of His-Purkinje system and carries ominous prognosis with high risk of complete heart block with slow (or no) escape rhythm.
We used to see this more often, back in the day…
Be careful here. This may look superficially like sinus but if you look in V1, you can see P waves just before the QRS (there is right bundle branch block (RBBB) and also P waves in the T wave (halfway in between) which are partly hidden. Atrial rate is about 150, ventricular rate 75. This is atrial tachycardia with 2:1 block (paroxysmal atrial tachycardia) "PAT with block" if it occurs paroxysmally) which may be due to digoxin toxicity, atrial disease, etc. If you got this right, PAT yourself on back; if you choked on it, a more forceful interscapular blow may be indicated.
 
The major finding is narrow complex tachycardia with atrial rate of about 300 indicating atrial flutter and slightly variable ventricular response (2:1 conduction, some 3:1 and probably some Wenckebach). Atrial flutter is sometimes mistaken (by housestaff at other institutions) for sinus tachycardia. Look, for instance, at V1 where the flutter waves are hard to see vs. lead II where they are more apparent. If only a V1 type lead were available, the mechanism could be readily mistaken.
 
Sinus tachycardia with 2:1 block (hidden P waves in T waves, e.g. leads V2 and V3) and left bundle branch block (LBBB). This most likely is type 2 block given its occurrence with sinus tachycardia and presence of LBBB. Patient was referred for evaluation for pacemaker. Remember the "meet me halfway" rule: always look halfway in between obvious P waves for hidden ones.
This one is tricky.  Be careful.
Note there is evidence here of mitral valvular disease:  ECG is highly suggestive of biventricular hypertrophy with left ventricular hypertrophy (LVH) type pattern in precordial leads and QRS axis of about +90 degrees. This combination is common with Rheumatic Heart Disease with MS/MR +/- aortic disease.  Rhythm is junctional with retrograde P waves with relatively long RP interval in lead II, V1, etc. with single ventricular premature beat.  But don’t stop there:  of note is long Q-T with prominent U waves raising question of drug effect/toxicity or hypokalemia. Patient was on a phenothiazine here. The ST-T changes are consistent with hypertrophy, drug effect, ischemia, etc. Patient was also on digoxin with low therapeutic level.
 
Atrial flutter.  Don't miss hidden atrial (F) wave just after QRS. This is atrial flutter with 2:1 conduction. Note that with atrial flutter there aren't P waves per se but atrial waves. So if you talk about atrial flutter do not say "the P waves...." since once you say P wave, the informed listener will probably assume it's not atrial fibrillation or atrial flutter. On the other hand, depending on whom you are talking to, this may not be an issue. Also note left ventricular hypertrophy (LVH) and left axis with possible prior inferior wall myocardial infarction (MI) here. Note that typical flutter waves have negative polarity in lead II, as is the case here.
 
This is highly suggestive of Parkinson's disease.  The ECG shows the classic pseudo atrial flutter/fib type waves associated with Parkinsonian tremor. The marked amplitude of the waves, absence of an irregular ventricular rate and subtle hidden true P waves in selected leads (lead 2, intermittently, aVR) indicate the diagnosis. Refer to neurology not cardiology.
 
This is sinus rhythmn.  This is a case of pseudo-flutter waves due to artifact (probably Parkinsonian tremor). Some leads show clear sinus P wave, e.g. V2, V3.  So, remember, look at all the leads.
 
AV nodal reentrant tachycardia (AVNRT). This tracing demonstrates a REGULAR narrow complex tachycardia at a rate of 150. The differential diagnosis of this supraventricular tachycardia includes: sinus tachycardia, AVNRT, automatic atrial tachycardia, atrio-ventricular reentrant tachycardia (AVRT), involving retrograde conduction over a "concealed" bypass tract, or atrial flutter with 2:1 block. If P waves can be located it can be helpful in determining the mechanism of the tachycardia. In this tracing P waves can be located at the end of the QRS in lead II and aVF, producing a "pseudo S" wave. Absence of P waves or a "pseudo S" pattern in II, III, or aVF, or a "pseudo R prime" pattern in V1 is characteristic of AVNRT due to near simultaneous activation of the atrium and the ventricle from the AV node. If the diagnosis is still in doubt, adenosine may be useful in slowing conduction in the AV node. Reentrant rhythms (AVNRT or AVRT) may "break" abruptly, converting to sinus; the ventricular response of other atrial tachcyardia or atrial flutter will slow often revealing underlying atrial activity. (Note: the final wide complex waveform is either an artifact or a ventricular premature beat.)
A Pause, for understanding...
Here is another AVNRT, with those pseudo-S waves:
But AVNRT gets confused with the less common AVRT:
AVRT is the consequence of the WPW-type congenital bypass tracks
The short PR and Delta Wave of WPW
A “classical” 12-lead EKG of WPW
So, what’s this?
Pre-admission testing showed pre-excitation, with subtle (very, very subtle if you missed it) Wolff-Parkinson-White (WPW) pattern. Note shortish PR and delta wave, best seen in V5 as slurring of initial QRS, with tall R (positive delta wave) in V1, negative in aVL, compatable with lateral bypass tract. What is differential diagnosis of tall R in V1? You should consider 1) normal variant, esp children, but also some adults, or lead placement; 2) funny chest/mediastinal configuration with heart more rightward; 3) posterior myocardial infarction (MI) (inferior/lateral ischemia/MI usually also seen); 4) right ventricular hypertrophy (usually right axis); 5) WPW with posterior or lateral bypass tract; 6) right ventricular conduction abnormality; 7) hypertrophic cardiomyopathy (HCM) with or without obstruction; 8) Duchenne muscular dystrophy (due to postero-basal fibrosis).
 
Atrial fibrillation with the Wolff-Parkinson-White (WPW) syndrome, with conduction down the bypass tract. This is for the most part a wide complex tachycardia with a rate of about 230 beats/min. The differential diagnosis includes 1) ventricular tachycardia, 2) supraventricular tachycardia with aberrancy, and 3) WPW with conduction down the bypass tract. The major clues include the "irregularly irregular" rhythm and the extremely rapid rate. Ventricular tachycardia may be mildy irregular but this degree of irregularity would be unusual at this very fast rate. The short refractory period of certain bypass tracts can allow extremely rapid heart rates, especially during atrial fibrillation. A correct diagnosis is very important because drugs that slow AV conduction (verapamil, beta blockers, digoxin, adenosine) are contraindicated. These drugs can facilitate preferential conduction down the bypass tract and the atrial fibrillation can degenerate to ventricular fibrillation.  A drug of choice is IV procainamide and if this is unsuccessful, DC cardioversion should be performed promptly.
 
Ectopic atrial (non-sinus) tachycardia with 2:1 block and 3:2 AV Wenckebach (beats 4 and 5). Note the negative P waves in lead II indicating a non-sinus pacemaker. Other important abnormalities: right bundle branch block, left anterior fascicular block and a markedly prolonged QT(U) interval. This patient was on digoxin and quinidine. The evidence for quinidine toxicity (assuming the serum potassium is normal and no other factors prolonging ventricular repolarization are identifiable) is the very long QT(U) interval best seen in V2-V5. This finding predisposes to torsade de pointes and may occur with "therapeutic" or "subtherapeutic" levels of quinidine or other class 1A antiarrhythmic drugs. Atrial tachycardia with Mobitz I block also strongly raises the question of digitalis toxicity. With pure 2:1 AV block, it may be impossible from the surface ECG to tell if the non-conducted beats are due to nodal or infranodal block. The clue here centers on beats 4 and 5 where 3:2 Wenckebach is evident, indicating block in the AV node.
 
It's only NSR if that stands for Not Sinus Rhythm. Note that there are two P's for every QRS (see V1) and that atrial activity at 176/min is negative in lead 2. This is atrial (not sinus) tachycardia with 2:1 block (conduction). Block is almost certainly at AV node level (Type 1 mechanism). Always exclude digitalis excess with this arrhythmia (not present here). ECG also shows low limb lead voltage, left ventricular hypertrophy (LVH) by precordial voltage and slow R wave progression
 
There is a wide-complex tachycardia with right bundle branch and left axis deviation (RBBB/LAD) pattern (old per hx). The rhythm is NOT sinus but atrial flutter with subtle flutter waves (aVR, II, V1). Note the negative polarity of atrial waves in lead II, characteristic of typical atrial flutter. Atrial flutter with 2:1 conduction is among most commonly missed major rhythms (often called sinus tachycardia by housestaff-- at other institutions, of course). Sustained atrial flutter is likely the major contributor, if not the precipitant of congestive heart failure syndrome here in this elderly man. Note that patients with atrial flutter may be candidates for ablation treatment to interrupt reentrant wave that underlies this rhythm.
 
Sinus rhythm with AV Wenckebach (Type 1 AV block) is present with 3:2 and 2:1 conduction. Notice progressive PR prolongation in 3:2 cycles with non-conducted sinus P wave followed by normal PR interval. Etiologies include drugs (beta-blockers, digitalis, calcium channel blockers), inferior ischemia, intrinsic AV node disease, Lyme disease, etc. In healthy young adults with high vagal tone (e.g. athletes) AV Wenckebach during sleeping hours is not uncommon. This patient's ECG also shows left ventricular hypertrophy (LVH), left atrial abnormality (LAA) (biphasic P wave with broad negative component in V1) and infero-lateral T wave inversions raising question of ischemia.
 
Atrial flutter with 2:1 conduction. Don't overlook subtle "extra" atrial wave (it's NOT a true P wave, but a flutter wave) in the early ST segment. Atrial flutter with 2:1 conduction (block) is often mistaken for sinus tachycardia or paroxysmal supraventricular tachycardia (PSVT)
 
There is a wide-complex tachycardia. The QRS shows a classic left bundle branch block (LBBB) pattern. If you look carefully, you will see atrial activity in the limb leads, with negative polarity in lead II, at rate of 320/min. Hence, atrial flutter with 2:1 conduction and LBBB.
 
It may look like sinus tachycardia, but look again in V1: There are two atrial waves for each QRS, so the atrial rate is 260. This is another case of atrial flutter with 2:1 conduction, one of the most commonly mistaken arrhythmias! This case is particularly tough because of low voltage in limb leads.
 
ECG shows sinus rhythm at 75/min but with 2:1 AV block. There is also left atrial abnormality (LAA). Conducted QRS complexes show right bundle branch block (RBBB) with left axis deviation, most c/w left anterior fascicular block. Minuscule R waves in II, III, aVF also raise question of prior inferior myocardial infarction (MI). The normal PR interval in conducted beats and the RBBB favor an infranodal location of block. Aficionados may also note the ventriculophasic sinus arrhythmia (P-P interval with QRS in-between is shorter than P-P without intervening QRS due to vagal tone fluctuations). Treatment: pacemaker.
 
Narrow complex tachyardia at 180/min with probable subtle retrograde P waves just after the QRS (may need to print this out to see in lead aVR and II). Therefore, likely diagnosis is AV nodal reentrant tachycardia (AVNRT); although, concealed bypass tract tachycardia or atrial tachycardia not excluded. This is, of course, NOT sinus and is also NOT atrial flutter (no flutter waves and ventricular rate is too fast for flutter with 2:1 conduction).
 
The underlying rhythm is atrial fibrillation (AF) with a slow regularized ventricular response (45/min) consistent with complete heart block (CHB) with a junctional escape mechanism. The QRS complexes show an intraventricular conduction delay (IVCD) with left axis and left ventricular hypertrophy (LVH) and there is Q-T(U) prolongation. Etiologic differential diagnosis includes intrinsic conduction disease, rule out drug toxicity or metabolic factors. The long QT here is against digitalis as sole factor (digitalis shortens the QT). Hypokalemia or relevant drug effects, such as sotalol or amiodarone, should be excluded. Treatment: If no reversible causes, patient will likely require an implanted ventricular pacemaker, as well as anticoagulation for AF.
 
Atrial tachycardia with 2:1 block and ventricular premature complexes (VPBs). Check out lead V1 if you missed this. The atrial rate is 200. (Points against atrial flutter are the atrial rate and discrete-appearing P waves, upright in lead 2). Also present: left ventricular hypertrophy (LVH) with intraventricular conduction defect (IVCD) (incomplete left bundle branch block-LBBB) and poor R wave progression (cannot rule out prior anterior myocardial infarction (MI); but this could be due to LVH/IVCD alone); also non-specific ST-T changes. The patient was not digitalis toxic.
Major Segue! Homework Assignment for the interns: Ten signs or symptoms with names derived from mythology.
Back to work:
This does indeed look like atrial flutter or coarse atrial fibrillation with regularized response. However, if you look closely, (V4; also lead 2 rhythm strip at end) you can see true P waves buried in baseline artifact. So, probable tremor artifact.
 
Atrial flutter with 2:1 block. Note that there are 2 atrial waves for each QRS in lead V1. The atrial rate is about 240; therefore, this could easily be misread as sinus tachycardia or paroxysmal supraventricular tachycardia (PSVT) variant--not by you of course. ECG also shows left ventricular hypertrophy (LVH) and possible prior antero-septal myocardial infarction (MI), along with non-specific ST-T changes.
 
Atrial fibrillation with an extremely rapid ventricular response (overall average about 204 beats per minute). There is intermittent right bundle branch block (RBBB) aberrancy (note QRS morphology in V1/V6, especially), associated with the very fast rates. In all, there are 10 consecutive beats with RBBB aberrancy, during which the ventricular response rate is up to 240 beats per minute
 
This looks like sinus bradycardia at a rate of 43 bpm, BUT... if you look more closely, there are P waves partly hidden in T waves. So this is actually pure sinus with blocked atrial premature complexes (APBs) in a bigeminal pattern. As a result the effective sinus rate is very slow. There are also non-specific T wave inversions noted in V1-V3.
 
The rhythm is sinus with long, atypical AV Wenckebach cycles (e.g., probable 6:5 block). The reason for the type 1 second degree AV block is apparent from the rest of the tracing which shows classic findings of an acute inferior/right ventricular myocardial infarction, consistent with proximal to mid-occlusion of the right coronary artery (RCA). The finding of ST elevation in lead III>II, as present here, is also consistent with this finding.
 
The rhythm is sinus with ventricular bigeminy and late diastolic premature ventricular complexes (PVCs) that come just after the second sinus P waves. The PVCs have a right bundle branch block (RBBB) morphology, but this is not intermittent RBBB or Wolf-Parkinson-White Syndrome (WPW).
 
The rhythm is sinus (or possibly atrial) tachycardia with a P wave rate of about 120/min with AV Wenckebach. There is an evolving inferior (posterior) myocardial infarction (MI) pattern with Q waves and T wave inversions in 2, 3, aVF, and a tall R in V2. The AV conduction disturbance is likely related to the MI. There is also low voltage and non-specific lateral ST-T changes.
 
Normal sinus rhythm with artifact in V4-V6, simulating premature ventricular complexes (PVCs). Note that simultaneous lead II (bottom) does not show this, and normal rhythm is not affected by the spurious "beats."
 
Classic Wolff-Parkinson-White (WPW) pre-excitation pattern, with triad of short PR, wide QRS and Delta waves. Note the polarity of the delta waves (most positive in lead 2 and lateral chest leads) consistent with a right-sided bypass tract. Patients with WPW may not only have atrio-ventricular reentrant tachycardia involving the bypass tract, but also AV nodal reentrant tachycardia, as well as atrial fibrillation (AF), etc.
One other segue -- here is WPW in reverse: That is, White-Parkinson-Wolff
 
Sinus with 5:4 AV Wenckebach. Low voltage (consistent with obesity, Chronic Obstructive Pulmonary Disease (COPD), and/or myocardial infarction (MI) with extensive cardiac muscle necrosis) makes identification of the very broad, low amplitude P waves difficult! Note subtle changes in T-wave morphology due to superimposed atrial activity. (Right axis deviation here could be due to COPD, lateral extension of infarct, or left posterior fascicular block.)
 
Normal sinus rhythm with a marked prolonged PR ("1st degree AV block"). The P waves, partly obscured by the T waves of the preceding sinus beats, are more clearly evident after the premautre ventricular complexes (PVCs), (beats 3 and 9, and the last beat). In addition, there is an intraventricular conduction delay (IVCD), borderline left axis deviation, and lateral ST-T wave changes associated with a history of multiple non-Q wave myocardial infarctions. The QT appears long in part because of the superimposed P wave deflection at the terminal portion of the T wave.
 
Sinus rhythm with atrial bigeminy (this pattern starts after the 4th sinus beat). In addition, there are voltage criteria for left ventricular hypertrophy (LVH), possible left atrial abnormality, and poor R-wave progression in V1-V3. The latter finding could be due to LVH, underlying anterior myocardial infarction (MI), lead placement; or could be a physiologic variant. The seventh QRS complex exhibits aberrancy, with a left anterior hemiblock morphology.
 
This is most likely atrioventricular reentrant tachycardia (AVRT), one of the "short R-P" tachycardias. Note the inverted P waves in leads II, III, and F; with upright P waves in aVR, consistent with retrograde activation of the atria from the region near the AV node and septum. Therefore, the descending limb of this reentrant rhythm is down the AV node, and the ascending limb is up a bypass tract located close to the septum. The differential diagnosis also includes AV nodal reentrant tachycardia (AVNRT) with a somewhat prolonged retrograde (fast pathway) limb, or, less likely, atrial tachycardia coming from low in the atria, with a very prolonged PR interval.
 
At first glance this might appear to be "sinus tachycardia." However, a second look reveals two interesting findings. Note the unusual P wave axis (negative in I and aVL) and the short PR interval (or very long RP interval). One possible explanation for these findings is that this is an atrial tachycardia with either 1) a rapidly conducting ("slick") AV node, or 2) an "atrio-His" bypass tract with fast conducting properties. This would account for the short PR interval in the presence of an atrial tachycardia. Another possibility for this non-sinus tachycardia is that it represents a "long RP tachycardia" involving a left lateral bypass tract which is slowly conducting (to account for the long RP interval). In this latter case, the reentrant circuit would involve antegrade (orthodromic conduction) down a rapidly conducting AV node and through the ventricular myocardium, with retrograde conduction up the slowly conducting bypass tract, and then across the atria back to the AV node.Note that resting heart rates above 140 beats/min are rarely due to sinus tachycardia in the very elderly.
 
The rhythm is atrial fibrillation with a rapid ventricular response (irregular rate at about 120 beats/min). There is an extremely wide QRS of left bundle branch block morphology; the QRS duration of over 200ms is suggestive of drug effect or hyperkalemia. Flecainide is a class Ic (Na+ channel blocking) drug, which exhibits "use dependency": at faster rates with less time for dissociation of the drug from the channel, Na+ channels are increasingly saturated with the drug. This increases the degree of QRS prolongation, and may be mistaken for ventricular tachycardia.
 
Ventricular tachycardia (VT) at rate of 170. The right bundle branch block morphology is atypical (monomorphic R, rather than rSR', in V1), and the R:S ratio is less than 1 in V6, both suggestive of ventricular tachycardia. Although the most common underlying diagnosis in adult North American patients with sustained monomorphic VT is coronary heart disease status post myocardial infarction (MI)s, this patient had a non-ischemic cardiomyopathy. The morphology of the VT is suggestive of origin from the left side of the heart, near the base (right bundle branch block with inferior/rightward axis). As with any wide complex tachycardia that is hemodynamically tolerated, the most important data to obtain is the 12-lead ECG, as this can direct possible radiofrequency ablation at the time of electrophysiology study.
(This is quite difficult, but worth it.)
Atrial flutter (negative atrial waves in II, positive in V1) with an atrial rate of approximately 230 BPM with subtle group beating. On close inspection, there are groups of two ventricular beats followed by pauses, with more atrial waves than there should be for simple 3:2 AV Wenckebach conduction pattern. This pattern is attributable to dual levels of block (multilevel block) in the AV node: in this case 2:1 AV conduction "above" 3:2 AV Wenckebach conduction. If you said atrial flutter (or raised question of flutter vs.rapid atrial tachycardia) along with Wenckebach variant, give yourself four out of five stars.
 
This is most consistent with accelerated idioventricular rhythmn (AIVR), originating from the left ventricle and therefore accounting for the atypical RBBB morphology. ST elevations in the precordial leads are likely due to the injury current from and underlying acute myocardial infarction (MI). AIVR may be marker of reperfusion following acute MI. The rate of about (83 per minute) is too slow for ventricular tachycardia and too fast for complete heart block.
 
This is relatively slow bi-directional ventricular tachycardia (VT). Note the sinus P waves (sinus tachycardia) at a rate of about 110 beats/min (most visible in the lead II rhythm strip toward end of recording) with apparent atrioventricular (AV) dissociation and a wide complex tachycardia at about 100 beats/min. The QRS complex alternates between two morphologies. This arrhythmia should always raise suspicion of digitalis toxicity (not present in this patient with a recent myocardial infarct). The name, bi-directional VT, is confusing, as are many terms generated by cardiologists, who in the main, are poorly literate in my estimation.  It refers to alternating right-and left- BBB conduction of the ectopic ventricular focus.  “Ventricular remodeling” is another term that drives me crazy; it evokes images of rearranging the furniture in the LV chamber.
 
Atrial fibrillation with a very rapid ventricular response (average 160-170/min).Other Findings: Low limb voltage, minor RV conduction delay with rsr' (or possibly Qr) in V1 complex, and non-specific ST-T changes (on digoxin).
 
Coarse atrial fibrillation (not flutter) is present with a slow ventricular response. There is an atypical left bundle branch block (LBBB) pattern. The rsR' in lateral leads (e.g., V6 here) is highly suggestive of severe left ventricular (LV )dysfunction, often associated with a ventricular aneurysm (El-Sherif sign). A QR or rsR' complex is also present in I and aVL. Left axis deviation and a long QT are noted, as well. The patient had heart failure due to prior silent infarction.
 
Atrial tachycardia (NOT sinus tachycardia) with variable block. Also left ventricular hypertrophy (LVH) and left axis deviation are present. This is not complete heart block either. Note that the rate is not slow and regular. Instead, the changing PR is due to variable AV block. This patient has a history of rheumatic heart disease (mitral and aortic regurgitation) and underwent a cardiac electrophysiology procedure with ablation of the ectopic atrial site that was the source of the tachycardia. Always exclude digoxin excess (not present here) in cases of atrial tachycardia with block as well. (Note: the subtle variation in QRS morphology, e.g., in leads II,IIII and aVF, which is due to varying degrees of superposition of the P waves on the QRS complexes and also, possibly, to slight changes in QRS conduction at different rates. This nonspecific finding should be distinguished from 2:1 electrical alternans patterns.)
Now let’s cover some  ventricular issues:
Causes of wide-QRS tachycardia
Brugada algorithm
Differential criteria considering the QRS morphology in leads V1 and V6 in case of right bundle branch block (RBBB) or left bundle branch block (LBBB) aspect.
Our first unknown. Rapid SVT or V. Tach?  Makes a difference, right?
We’ll start with the Brugada algorithm
Are there any RS complexes in the V leads?  Absence of any RS complex in the precordial leads V1..V6 is 100% specific for VT, but is insensitive (26%).
Here’s the tracing once again. So, is there an RS in any of the precordial leads?
Yes, V2 has an RSR’. So let’s go to the next step of the Brugada Algorithm...
Brugada algorithm
In  any  of these RS complexes, is the interval from start of R to nadir of S > 100ms ?
Look at V2, and specifically at the fifth complex there... The R wave starts on a solid line, and the nadir of the S wave  is only two small boxes or .08 secs (80 msecs) out.
So we need to go to the next step in the algorithm...
Is there AV dissociation, or are there fusion or capture beats?  {A capture beat QRS looks normal because a conducted sinus beat transiently captures the ventricles;
 fusion beat's QRS is intermediate between normal and the VT complex}
Well, what do you think? I think I can see P waves.  Can you?
So, if we are correct, then according to the algorithm this is V. Tach, because there are P waves not associated with any QRS’s.  But let’s suppose we aren’t sure.  Let’s go to the next step of the B.A., that is examining morphology.  (This is tedious, but productive.) Is the QRS complex in V1 predominantly positive?
No it isn’t.  So this doesn’t help us. Let’s move on...
The RS interval is over 60 ms., so...
Next look at V6. Is there a monophasic R  ,  or  a QR  ,  or  an R:S ratio < 1  ,  or  a QS? If any of these morphologic criteria exist, it’s likely to be V tach.  What’s your call?
This is a rapid monomorphic ventricular tachycardia (VT), 280 beats per minute, associated with hemodynamic collapse. This tracing was obtained from a patient with severe ischemic cardiomyopathy during an electrophysiologic (EP) study. The rhythm later converted to sinus with a single external shock. This patient had an atrial rate of 72 beats per minute (measured with intracardiac electrodes, not shown). Although ventriculoatrial dissociation (faster V rate than A rate) is diagnostic of VT, the surface ECG findings are only present approximately 20% of the time. In this tracing, the ventricular rate is simply too fast for P waves to be observed. VT with cycle lengths from 200-240 ms is often termed ventricular flutter. So by morphology, and because we think We see A-V dissociation as well, we can say:
But here’s an important point: if there is hemodynamic collapse, shoot first and ask questions later: Often the best 'drug' is in fact electricity, as sustained VT often deteriorates into VF if left unmanaged.
Okay, next case: Is this supraventricular, or nonsustained V. Tach?
If we use the  algorithm...
You will readily see that two sensitive criteria for V. Tach are met: There is A-V dissociation, and in V1, the interval from the start of the R wave to the nadir of the S wave is > 100 msec.
Can you see the P waves, without corresponding QRS’s? (A-V dissociation) And in V1, the first complex starts on a solid Line and the nadir of the S wave is at least 2.5 boxes later (i.e. .10 sec. or 100 msec)
So this is repetitive monomorphic ventricular tachycardia (VT) from an asymptomatic 45-year-old female wind surfer. This patient has a structurally normal heart. This ECG pattern is typical for a form of idiopathic VT arising from the right ventricular outflow tract. Unlike ischemic VTs, this form of VT is often provoked by exercise and suppressed by beta blockade or verapamil. The prognosis for these patients is good, with the following 2 exceptions: (1) occasionally, this form of VT is associated with right ventricular dysplasia, which is associated with sudden death, and (2) occasionally, patients with incessant VT develop congestive heart failure due to tachycardia-induced cardiomyopathy. The cardiomyopathy generally resolves when the tachycardia is treated.
What’s this one?
You might be tempted to jump to “atrial fibrillation with an aberrated ventricular conduction” but let’s use the algorithm first.
Brugada algorithm
There are RS’s in the precordial leads, so that criteria is not met, those are very broad S waves (not to be confused with inverted broad ST/T segments) so the RS interval is certainly greater than 100 msec.
Agree? Also, look at V4-6.  Remind you of anything?
This ECG shows a relatively regular, wide complex polymorphic ventricular tachycardia, rate 150 bpm, that is most likely a type of torsade(s) de pointes. See leads V4-V6 for classic “turning of the points” morphology. Intermittent ventricular fusion beats may be present. Causes of this life-threatening arrhythmia include a variety of drugs and specific electrolyte abnormalities (especially hypokalemia and hypomagnesemia) associated with long QT (U) intervals, as well as hereditary long QT syndromes due to “channelopathies.”Admission laboratory testing revealed a serum potassium of 2.2 (normal range 3.5-5.3 mEq/L), sodium of 143 (range 135-148 mEq/L), bicarbonate of 16 (normal 21-30mEq/L), phosphate 1.0 (range 2.7-4.5 mg/dL) and magnesium 2.5 (range 1.6-2.6 mg/dL.) Polymorphic V. Tach
This one should be easy for you now...
Because there are RS waves in the precordial leads, that criteria is not met, but the RS interval is huge. Let’s use that other table of criteria on this one, just for fun...
Differential criteria considering the QRS morphology in leads V1 and V6 in case of right bundle branch block (RBBB) or left bundle branch block (LBBB) aspect .
Do you agree that this looks RBBB-like?
Differential criteria considering the QRS morphology in leads V1 and V6 in case of right bundle branch block (RBBB) or left bundle branch block (LBBB) aspect. So, the tracing has a RBBB like  appearance, and V1 has a qRSR’.
So this one is monomorphic V. Tach The ECG shows a relatively regular, very wide-complex tachycardia with a heart rate of 130/min and a QRS duration of about 180 ms. The QRS morphology is that of a right bundle branch block (RBBB) with a qRSR′in V1, rS in V6 and a marked rightward axis. Underlying atrial activity is hard to discern with certainty but may be sinus with AV dissociation. These factors indicate ventricular tachycardia which was confirmed at cardiac electrophysiology study. A left ventricular tachycardia intramyocardial focus (associated with this ECG pattern) localized below the anterior papillary muscle was successfully eliminated with radiofrequency ablation.
A wide-complex tachycardia. V. Tach?
Let’s start with this again...
There are RS’s in the precordial leads, so that’s out. The RS interval is not greater than 100 msec. so that’s out too.
Is there A-V dissociation? Actually, there is A-V  ass ociation; Look at lead III
So now we have to go through all those laborious morphologic criteria: Is the QRS complex in V1 predominantly positive?
Yes it is. So:
V1 is a QR So, next look at V6...
Ans.: none of the above, so this is SVT, but...
A close look at V5 gives us another way to attack this...
So this is not V. Tach.  This is Atrial flutter with 2:1 conduction and RBBB This ECG shows subtle atrial flutter with 2:1 block, right bundle branch block and right axis deviation. The atrial rate is about 260 bpm (see upright flutter waves in lead V1) with a ventricular response rate of about 130 bpm. The right axis deviation here, in the absence of clinical evidence of right ventricular hypertrophy or lateral infarction, likely represents left posterior fascicular block.The patient had persistent atrial flutter with variable block or atrial fibrillation at other times.
Here’s a three-fer.  Careful.
This ECG shows underlying coarse atrial fibrillation (AF) with (right) ventricular demand pacing at a rate about 78 bpm with a relatively wide QRS complex (left bundle branch block morphology) at about 180 milliseconds. Two PVCs are seen (2nd and 6th beat).There are no sinus P waves or flutter waves and no atrial spikes to indicate dual chamber pacing.  You don’t need algorithms for this one.
What is this?
Back to the algorithm...
My problem is knowing where the baseline is in the precordial leads, and therefore defining R, S, and Q waves. So we’ll need to do the morphology thing again. Or use that other chart...
Differential criteria considering the QRS morphology in leads V1 and V6 in case of right bundle branch block (RBBB) or left bundle branch block (LBBB) aspect.
The rSR’ in V1 suggests a RBBB
...and the QS in V6 gives it away...
This is monomorphic ventricular tachycardia at a rate of about 140 bpm with a very wide (QRS duration about 0.20 second) with a right bundle branch block and superior axis deviation morphology.  The previous ECG done on this patient the same day showed an extensive anterolateral ST elevation acute myocardial infarction .  There is suggestion of ST elevation here in V5 and V6 which is difficult to interpret with the wide QRS here.
An easy one...
This ECG shows sinus rhythm with ventricular bigeminy (every other beat is a ventricular premature beat or complex (VPB or VPC, for short) of the same morphology in each lead.Note also the minimal diffuse ST elevations (with non-specific T wave inversions). These repolarization changes are consistent with sub-clinical perioperative pericarditis, a ECG common finding after coronary artery bypass graft (CABG) surgery.
Try this:
The problem here is that it’s impossible for me to fit this into a BBB category (and use the table), and I can’t really define the baseline well enough to use the Brugada algorithm. So let’s see if we can dissect this out...
This ECG shows a very wide-complex (between 160-200 msec) tachycardia at a rate about 180 bpm with a bundle branch (BBB) morphology.
The diagnosis of monomorphic ventricular tachycardia is strongly supported by the following criteria:  1) Wide QRS (greater than 140 ms) in the absence of drugs or hyperkalemia (the QRS here is extremely wide);  2) wide monophasic R wave in lead V1 (or RS or QR complex in that lead (or whatever we choose to call it).
3) QR complexes in the inferior leads with extreme axis deviation;  4) rS or QS waves in V6. It’s the rS in V6 that makes me worry about V. tach.
A resting rate of 180 bpm excludes sinus tachycardia, especially in this age group. There is no evidence of atrial flutter or atrial tachycardia, and, as described, the QRS morphology here is strongly against aberrant supraventricular conduction. This patient had a complicated cardiac history of coronary artery disease, status post bypass graft surgery and prior inferior infarction. He had recurrent ventricular tachycardia, likely originating the area of the prior inferior infarct. An echocardiogram revealed showed a dilated left atrium and left ventricle, extensive thinning with an akinetic scar of the inferior base and moderately severe mitral regurgitation. Estimated left ventricular ejection fraction was 35%.
This is a great tracing.  I hope you’re saving these.
Let’s see if our algorithm works for this one:
There are RS complexes in the precordium, but the RS interval is quite long, much greater than 100 msec, so that criteria is fulfilled for V. Tach
Now let’s use the table.  Is this more RBBB or LBBB?
LBBB type.  Here’s the table: The R in V1 is not greater than  30 ms, and V6  has an RS, so neither criteria is fulfilled here for V tach .
So we have conflicting algorithms.  Is there anything else that helps us here?  Think outside the box:
The first 4 beats of this ECG show termination of a run of wide-complex tachycardia at rate about 150 bpm. The 5th and the 6th beats supraventricular conduction with underlying atrial fibrillation now apparent. The conducted supraventricular beats suggest underlying inferior myocardial infarction with prominent Q waves in leads II and aVF and a borderline intraventricular conduction delay (QRS=110 msec). Starting with the 7th beat and continuing to the end, the wide-complex ventricular tachycardia resumes. The wide-complex tachycardia (QRS about 160 msec) has left bundle branch block morphology, with an inferior QRS axis.This ECG shows two features strongly favoring ventricular tachycardia over supraventricular tachycardia with left bundle aberrancy, namely: 1) a broad R wave (40 msec or more in duration) in leads V2, and 2) a delayed intrinsicoid deflection great than 70 msec in V1-V2 measured from the initial onset of the QRS to the nadir of the S wave. (The absence of these findings does not exclude ventricular tachycardia, however.)This patient ruled in for a small acute myocardial infarction with an elevated troponin, but normal creatine kinase. Echocardiogram showed a major infero-lateral wall motion abnormality consistent with prior myocardial infarction as well. He had recurrent episodes of ventricular tachycardia and underwent successful ablation of an ectopic focus in the crest of the interventricular septum, under the left coronary cusp, consistent with the monomorphic ventricular tachycardia seen here with a left bundle branch block, inferior QRS axis morphology.
New case:  what do you think?
For the sake of brevity, I will tell you that neither the Brugada algorithm nor the table work for this one.  That is to say, both will tell us that this is SVT. Well then, what kind of SVT?  It’s not flutter.  There are no flutter waves.  Nor are there any atrial fibrillatory waves.  It’s not WPW; there are no delta waves.  There are no reentrant atrial waves seen, although after some of the ventricular complexes, one can see P waves. Sometimes the algorithms mislead us.  That is why you need to think for yourself.  This is hard stuff, but no one can fault you for saying: “I’m still worried about V. Tach.”
This ECG shows sinus rhythm (normal QTc) with runs of non-sustained monomorphic ventricular tachycardia (starting 2nd beat, rate 170 bpm). The ventricular beats have a left bundle branch block pattern and inferior/borderline rightward QRS axis raising consideration of repetitive monomorphic ventricular tachycardia arising from the right ventricular outflow tract (RVOT). Isolated premature ventricular complexes (PVCs) and two ventricular couplets are also present with the same morphology.Atrial tachycardia is excluded as there are no premature P waves prior to the runs. The P waves associated with the wide complex beats appear to be retrograde consistent with ventricular-atrial conduction. No atrial flutter waves are seen. There is no delta wave to suggest Wolff-Parkinson-White pattern.The patient had an idiopathic cardiomyopathy with global mild ventricular hypokinesis. Previous attempts at ablation in the RVOT region had been attempted. At the time of repeat cardiac electrophysiologic study, the arrhythmia was not inducible, despite intensive programmed electrical and pharmacologic stimulation. The diagnosis was monomorphic ventricular tachycardia of unknown mechanism, possibly arising from the superior portion of the interventricular septum.
Trouble?
This is a classic ECG obtained during a ventricular fibrillation (VF) cardiac arrest. The findings are typical of ventricular fibrillation with more organized ventricular electrical activity in the second half of the strip consistent with ventricular flutter.There is no evidence of any atrial activity ruling out atrial fibrillation with Wolff-Parkinson-White (WPW ) or atrial flutter with a bundle branch block. The characteristic, systematic variation in QRS axis of torsade is not seen here (although torsade may degenerate terminally into ventricular fibrillation.)
This is one for the table:
The tracing has a classic RBBB configuration And is triphasic both in V1 and V6
There is a classic right bundle branch block (RBBB) morphology (rsR '  in V1) making VT unlikely. No definite atrial activity is seen (P waves or flutter waves). The very regular rate at 150/min excludes AF. Therefore, the rhythm is most consistent with paroxysmal supraventricular tachycardia (PSVT), most likely AV nodal re-entrant tachycardia (AVNRT), although AV re-entrant tachycardia or atrial tachycardia (with hidden P waves) are not excluded.
An easy one...
There are fixed coupled premature ventricular compexes (PVCs) in a bigeminal pattern. Note the onset of each VPC comes just after a (non-conducted) sinus P wave, giving the appearance of a short PR.The sinus beats show probable left atrial abnormality, borderline left axis deviation, and a minor right ventricular (RV) conduction delay .
Why the wide-complex rhythmn?
Both atrial and ventricular pacemaker spikes are seen, for example, in lead II, before each P wave and QRS, respectively. The patient has a dual chamber rate responsive pacemaker (DDDR) pacemaker.
A difficult one...
Complex ECG showing:  1) Underlying atrial fibrillation;  2) Long Q-T(U);  3) Ventricular pacing with intermittent sensing failure; and  4) Run of torsade de pointes-type of polymorphic ventricular tachycardia (second half of record).
Very tough.  Be careful.
Hint:  I showed you this one to illustrate the A-V dissociation criteria of the Brugada algorithm Do you see the A-V dissociation?
There is a wide complex tachycardia at about 165/min with an right bundle branch block (RBBB) morphology. Underlying sinus tachycardia at about 136/min and AV dissociation are noted, confirming the diagnosis of ventricular tachycardia (VT). (Note the clear sinus P waves, e.g., just after 5th QRS.) The QRS morphology and axis are consistent with origin of the VT from the posterior septum. The patient had had a prior large inferior-posterior myocardial infarction (MI). (Of additional note here is the relatively low limb lead voltage and sinus tachycardia).
Thoughts?
What helps me here is V6.  I get an idea of where the baseline is, and the RS interval here clearly exceeds 100 msecs.
Ventricular tachycardia (VT) at rate of 170. The right bundle branch block morphology is atypical (monomorphic R, rather than rSR', in V1), and the R:S ratio is less than 1 in V6, both suggestive of ventricular tachycardia. Although the most common underlying diagnosis in adult North American patients with sustained monomorphic VT is coronary heart disease status post myocardial infarction (MI)s, this patient had a non-ischemic cardiomyopathy. The morphology of the VT is suggestive of origin from the left side of the heart, near the base (right bundle branch block with inferior/rightward axis). As with any wide complex tachycardia that is hemodynamically tolerated, the most important data to obtain is the 12-lead ECG, as this can direct possible radiofrequency ablation at the time of electrophysiology study.
Is this ventricular tachycardia?
No. The rhythm is atrial fibrillation with a rapid ventricular response (irregular rate at about 120 beats/min). There is an extremely wide QRS of left bundle branch block morphology; the QRS duration of over 200ms is suggestive of drug effect or hyperkalemia. Flecainide is a class Ic (Na+ channel blocking) drug, which exhibits &quot;use dependency&quot;: at faster rates with less time for dissociation of the drug from the channel, Na+ channels are increasingly saturated with the drug. This increases the degree of QRS prolongation, and may be mistaken for ventricular tachycardia. The take-home message here, for this impossible case, is to ask what meds the patient is on.  Even if I were told “flecainide for A. Fib” I would not send this patient home.
Is this polymorphic V-tach?
No.  This is atrial fibrillation with an extremely rapid ventricular response (overall average about 204 beats per minute). There is intermittent right bundle branch block (RBBB) aberrancy (note QRS morphology in V1/V6, especially), associated with the very fast rates. In all, there are 10 consecutive beats with RBBB aberrancy, during which the ventricular response rate is up to 240 beats per minute (Both the algorithm and the table work well on this one.)
DC countershock immediately?
No.  This is a ventricular paced rhythm with underlying complete heart block (P waves march through). Most important is superimposed current of injury with hyperacute ST-T changes inferiorly and laterally with reciprocal change V1-V3 consistent with acute infero-postero-lateral ischemia/ myocardial infarction (MI). Pacemaker patterns, like left bundle branch block (LBBB), often mask acute or chronic MI. But sometimes, as in the present case, the ischemic changes &quot;show through.&quot;
Think this one through with me. Forget about algorithms.
Atrial fibrillation with the Wolff-Parkinson-White (WPW) syndrome, with conduction down the bypass tract. This is for the most part a wide complex tachycardia with a rate of about 230 beats/min. The differential diagnosis includes 1) ventricular tachycardia, 2) supraventricular tachycardia with aberrancy, and 3) WPW with conduction down the bypass tract. The major clues include the &quot;irregularly irregular&quot; rhythm and the extremely rapid rate. Ventricular tachycardia may be mildy irregular but this degree of irregularity would be unusual at this very fast rate. The short refractory period of certain bypass tracts can allow extremely rapid heart rates, especially during atrial fibrillation. A correct diagnosis is very important because drugs that slow AV conduction (verapamil, beta blockers, digoxin, adenosine) are contraindicated. These drugs can facilitate preferential conduction down the bypass tract and the atrial fibrillation can degenerate to ventricular fibrillation. Call EP fellows stat! A drug of choice is IV procainamide and if this is unsuccessful, DC cardioversion should be performed promptly.
Summary: Many of these are very difficult. The algorithms are hard to commit to memory. For family medicine practitioners who see this stuff infrequently, this can be an impossible situation,  unless you remember this axiom:
Always assume the patient has the worst treatable condition you can do something about, and proceed from there. In the case of wide-complex tachycardias, to say: “This is probably just SVT with aberrated conduction” and send the patient home, is to court disaster.
You do not have to know everything to be a great doctor. You just have to think.

Más contenido relacionado

La actualidad más candente

Eps basics,part2(lecture)
Eps basics,part2(lecture)Eps basics,part2(lecture)
Eps basics,part2(lecture)salah_atta
 
Electrophysiologic basis part3
Electrophysiologic basis part3Electrophysiologic basis part3
Electrophysiologic basis part3salah_atta
 
SUPRAVENTRICULAR TACHYCARDIA - SVT
SUPRAVENTRICULAR TACHYCARDIA - SVTSUPRAVENTRICULAR TACHYCARDIA - SVT
SUPRAVENTRICULAR TACHYCARDIA - SVTPinkesh Parmar
 
Differentiation between AVNRT and AVRT_advanced lecture
Differentiation between AVNRT and AVRT_advanced lectureDifferentiation between AVNRT and AVRT_advanced lecture
Differentiation between AVNRT and AVRT_advanced lectureTaiwan Heart Rhythm Society
 
Electrophysiology study protocol
Electrophysiology study protocolElectrophysiology study protocol
Electrophysiology study protocolSatyam Rajvanshi
 
ventricular tachycardia (VT) Localisation
ventricular tachycardia (VT) Localisationventricular tachycardia (VT) Localisation
ventricular tachycardia (VT) LocalisationMalleswara rao Dangeti
 
Wide complex tachycardia
Wide complex tachycardiaWide complex tachycardia
Wide complex tachycardiaAmir Mahmoud
 

La actualidad más candente (20)

Eps basics,part2(lecture)
Eps basics,part2(lecture)Eps basics,part2(lecture)
Eps basics,part2(lecture)
 
Ecg
EcgEcg
Ecg
 
11.atrial flutter for basic ep.final
11.atrial flutter for basic ep.final11.atrial flutter for basic ep.final
11.atrial flutter for basic ep.final
 
Electrophysiologic basis part3
Electrophysiologic basis part3Electrophysiologic basis part3
Electrophysiologic basis part3
 
SVT maneuvers
SVT maneuversSVT maneuvers
SVT maneuvers
 
Electrophysiologic Study
Electrophysiologic StudyElectrophysiologic Study
Electrophysiologic Study
 
ECG: Trifascicular Block
ECG: Trifascicular BlockECG: Trifascicular Block
ECG: Trifascicular Block
 
SUPRAVENTRICULAR TACHYCARDIA - SVT
SUPRAVENTRICULAR TACHYCARDIA - SVTSUPRAVENTRICULAR TACHYCARDIA - SVT
SUPRAVENTRICULAR TACHYCARDIA - SVT
 
Atrial tachycardia
Atrial tachycardiaAtrial tachycardia
Atrial tachycardia
 
WIDE QRS TACHYCARDIA
WIDE  QRS TACHYCARDIAWIDE  QRS TACHYCARDIA
WIDE QRS TACHYCARDIA
 
9.avnrt chang sl-0324-2
9.avnrt chang sl-0324-29.avnrt chang sl-0324-2
9.avnrt chang sl-0324-2
 
SVT-Alogarythm
SVT-AlogarythmSVT-Alogarythm
SVT-Alogarythm
 
ICD troubleshooting
ICD troubleshootingICD troubleshooting
ICD troubleshooting
 
Differentiation between AVNRT and AVRT_advanced lecture
Differentiation between AVNRT and AVRT_advanced lectureDifferentiation between AVNRT and AVRT_advanced lecture
Differentiation between AVNRT and AVRT_advanced lecture
 
Electrophysiology study protocol
Electrophysiology study protocolElectrophysiology study protocol
Electrophysiology study protocol
 
Supraventricular tacchycardias
Supraventricular tacchycardias Supraventricular tacchycardias
Supraventricular tacchycardias
 
ventricular tachycardia (VT) Localisation
ventricular tachycardia (VT) Localisationventricular tachycardia (VT) Localisation
ventricular tachycardia (VT) Localisation
 
Wide complex tachycardia
Wide complex tachycardiaWide complex tachycardia
Wide complex tachycardia
 
Essentials of ecg interpretation aphrs
Essentials of ecg interpretation aphrsEssentials of ecg interpretation aphrs
Essentials of ecg interpretation aphrs
 
Complex svt with differentiation
Complex svt  with differentiationComplex svt  with differentiation
Complex svt with differentiation
 

Similar a Arrhythmias July 09

Ekg Cases 7 15 09 Level 2 Part 1
Ekg Cases 7 15 09 Level 2 Part 1Ekg Cases 7 15 09 Level 2 Part 1
Ekg Cases 7 15 09 Level 2 Part 1Michael LaCombe
 
Analyze an Electrocardiogram
Analyze an ElectrocardiogramAnalyze an Electrocardiogram
Analyze an ElectrocardiogramAyesha Bukhari
 
ECG approach to arrhythmias 2017
ECG approach to arrhythmias 2017ECG approach to arrhythmias 2017
ECG approach to arrhythmias 2017Ashutosh Pakale
 
Atrial flutter and bradycardias
Atrial flutter  and bradycardiasAtrial flutter  and bradycardias
Atrial flutter and bradycardiasDR RML DELHI
 
THE WOLFF–PARKINSON–WHITE.pptx
THE WOLFF–PARKINSON–WHITE.pptxTHE WOLFF–PARKINSON–WHITE.pptx
THE WOLFF–PARKINSON–WHITE.pptxLatifaomary
 
Avn bradycardia mediated
Avn bradycardia mediatedAvn bradycardia mediated
Avn bradycardia mediatedAhmed Taha
 
Systematic approach to wide qrs tachycardia
Systematic approach to wide qrs tachycardiaSystematic approach to wide qrs tachycardia
Systematic approach to wide qrs tachycardiasalah_atta
 
ECG Basics with Dr LK Meher.pptx
ECG Basics with Dr LK Meher.pptxECG Basics with Dr LK Meher.pptx
ECG Basics with Dr LK Meher.pptxSubha Deep
 
Fascicular vt chetan new
Fascicular vt chetan newFascicular vt chetan new
Fascicular vt chetan newChetan Bhole
 
ECG Basics with Prof Dr LK Meher.pptx
ECG Basics with Prof Dr LK Meher.pptxECG Basics with Prof Dr LK Meher.pptx
ECG Basics with Prof Dr LK Meher.pptxSubha Deep
 
Tachyarrhythmias questions. by st jude. arranged by prof. samir rafla
Tachyarrhythmias questions. by st jude.  arranged by prof. samir raflaTachyarrhythmias questions. by st jude.  arranged by prof. samir rafla
Tachyarrhythmias questions. by st jude. arranged by prof. samir raflaAlexandria University, Egypt
 
ecg_systemic_approach_12-lead_compressed.pdf
ecg_systemic_approach_12-lead_compressed.pdfecg_systemic_approach_12-lead_compressed.pdf
ecg_systemic_approach_12-lead_compressed.pdfjiregnaetichadako
 
Ekg Cases 7 8 09 Level 1 Part2
Ekg Cases 7 8 09 Level 1 Part2Ekg Cases 7 8 09 Level 1 Part2
Ekg Cases 7 8 09 Level 1 Part2Michael LaCombe
 
A Guide TO ECG Interpretation
A Guide TO ECG InterpretationA Guide TO ECG Interpretation
A Guide TO ECG Interpretationmeducationdotnet
 
Beginners Guide for ECG Interpretation
Beginners Guide for ECG InterpretationBeginners Guide for ECG Interpretation
Beginners Guide for ECG Interpretationmeducationdotnet
 
Supra ventri cular arrhythmia 2021
Supra ventri cular arrhythmia  2021Supra ventri cular arrhythmia  2021
Supra ventri cular arrhythmia 2021rajasthan govt
 

Similar a Arrhythmias July 09 (20)

Ekg Cases 7 15 09 Level 2 Part 1
Ekg Cases 7 15 09 Level 2 Part 1Ekg Cases 7 15 09 Level 2 Part 1
Ekg Cases 7 15 09 Level 2 Part 1
 
ECG Rhythm Abnormalities
ECG Rhythm AbnormalitiesECG Rhythm Abnormalities
ECG Rhythm Abnormalities
 
Analyze an Electrocardiogram
Analyze an ElectrocardiogramAnalyze an Electrocardiogram
Analyze an Electrocardiogram
 
ECG approach to arrhythmias 2017
ECG approach to arrhythmias 2017ECG approach to arrhythmias 2017
ECG approach to arrhythmias 2017
 
Atrial flutter and bradycardias
Atrial flutter  and bradycardiasAtrial flutter  and bradycardias
Atrial flutter and bradycardias
 
THE WOLFF–PARKINSON–WHITE.pptx
THE WOLFF–PARKINSON–WHITE.pptxTHE WOLFF–PARKINSON–WHITE.pptx
THE WOLFF–PARKINSON–WHITE.pptx
 
Avn bradycardia mediated
Avn bradycardia mediatedAvn bradycardia mediated
Avn bradycardia mediated
 
Systematic approach to wide qrs tachycardia
Systematic approach to wide qrs tachycardiaSystematic approach to wide qrs tachycardia
Systematic approach to wide qrs tachycardia
 
ECG Basics with Dr LK Meher.pptx
ECG Basics with Dr LK Meher.pptxECG Basics with Dr LK Meher.pptx
ECG Basics with Dr LK Meher.pptx
 
Fascicular vt chetan new
Fascicular vt chetan newFascicular vt chetan new
Fascicular vt chetan new
 
Cardiac Arrythmias
Cardiac ArrythmiasCardiac Arrythmias
Cardiac Arrythmias
 
ECG Basics with Prof Dr LK Meher.pptx
ECG Basics with Prof Dr LK Meher.pptxECG Basics with Prof Dr LK Meher.pptx
ECG Basics with Prof Dr LK Meher.pptx
 
Tachyarrhythmias questions. by st jude. arranged by prof. samir rafla
Tachyarrhythmias questions. by st jude.  arranged by prof. samir raflaTachyarrhythmias questions. by st jude.  arranged by prof. samir rafla
Tachyarrhythmias questions. by st jude. arranged by prof. samir rafla
 
ecg_systemic_approach_12-lead_compressed.pdf
ecg_systemic_approach_12-lead_compressed.pdfecg_systemic_approach_12-lead_compressed.pdf
ecg_systemic_approach_12-lead_compressed.pdf
 
Ekg Cases 7 8 09 Level 1 Part2
Ekg Cases 7 8 09 Level 1 Part2Ekg Cases 7 8 09 Level 1 Part2
Ekg Cases 7 8 09 Level 1 Part2
 
Cardiac arrhythmias y2 oct 2010
Cardiac arrhythmias y2 oct 2010Cardiac arrhythmias y2 oct 2010
Cardiac arrhythmias y2 oct 2010
 
A Guide TO ECG Interpretation
A Guide TO ECG InterpretationA Guide TO ECG Interpretation
A Guide TO ECG Interpretation
 
Beginners Guide for ECG Interpretation
Beginners Guide for ECG InterpretationBeginners Guide for ECG Interpretation
Beginners Guide for ECG Interpretation
 
Supra ventri cular arrhythmia 2021
Supra ventri cular arrhythmia  2021Supra ventri cular arrhythmia  2021
Supra ventri cular arrhythmia 2021
 
Avrt vz vt
Avrt vz vtAvrt vz vt
Avrt vz vt
 

Más de Michael LaCombe

Más de Michael LaCombe (9)

Chest Lungs 3
Chest Lungs 3Chest Lungs 3
Chest Lungs 3
 
Chest Film Part 2
Chest Film Part 2Chest Film Part 2
Chest Film Part 2
 
Chest xrays pneumonias
Chest xrays pneumoniasChest xrays pneumonias
Chest xrays pneumonias
 
Chest Film Part 1
Chest Film Part 1Chest Film Part 1
Chest Film Part 1
 
Ekg Cases Jul09.Level One Part 1
Ekg Cases Jul09.Level One Part 1Ekg Cases Jul09.Level One Part 1
Ekg Cases Jul09.Level One Part 1
 
Acls Fmi 2009
Acls Fmi 2009Acls Fmi 2009
Acls Fmi 2009
 
Four causes of CHF09
Four causes of CHF09Four causes of CHF09
Four causes of CHF09
 
May 6, 2009
May 6, 2009May 6, 2009
May 6, 2009
 
April 8, 09 Ppt.
April 8, 09 Ppt.April 8, 09 Ppt.
April 8, 09 Ppt.
 

Arrhythmias July 09

  • 1. Arrhythmias M. LaCombe MDFMR/UNECOM July 22, 2009
  • 2. Let’s start with an easy one:
  • 3. Sinus rhythm with 2:1 AV heart block. The ECG shows a bradyarrhythmia with non-conducted sinus P waves alternating with normally conducted P waves. It is not possible to reliably identify the point of block (nodal vs. infranodal) from this single ECG with 2:1 conduction. There is no evidence of acute inferior ischemia, either. The site of block could be proximal (in the AV node) or more distal, in the His-Purkinje system. In general, with 2:1 block, involvement of the AV node is favored by a narrow QRS complex and a prolonged PR interval, or by the presence of intermittent AV Wenckebach. Block (infranodal) in the His-Purkinje system would be favored by a concomitant bundle branch block and/or with a PR interval of 160 ms or less. A possibly useful bedside diagnostic test for chronic 2:1 block (in the absence of active ischemia) would be to increase the sinus rate (mild exercise). A resumption of 1:1 conduction favors AV node block while worsening of block strongly favors infranodal disease. Pacemaker placement is indicated for symptomatic 2:1 block without reversible cause (e.g., drug effect) and generally for asymptomatic 2:1 block due to infranodal disease. Intracardiac His bundle electrogram would definitively identify the site of block. This patient had intermittent 3:2 AV Wenckebach at other times, and then resumed 1:1 conduction on subsequent ECGs, consistent with AV node disease
  • 4. Put this one together for me:
  • 5. Sinus rhythm with AV Wenckebach with 4:3 conduction in the setting of an acute inferior wall infarction. The ECG demonstrates Q waves and ST elevation in leads 2, 3, and aVF. There are also reciprocal ST segment depressions in leads 1, aVL and V2-3. The rhythm is Wenckebach showing progressive prolongation of the PR intervals, shortening of the R-R intervals and block of every fourth P wave. The presence of &quot;group&quot; beating is easily recognized and characteristic of Wenckebach and is often associated with high vagal tone or nodal ischemia in the setting of an inferior wall myocardial infraction (MI). The block is at the level of the AV node
  • 6. You’ve seen this one before, but now, let’s focus on the rhythm
  • 7. Atrial fibrillation with a rapid ventricular response. Right axis deviation. Intraventricular conduction delay. Right ventricular hypertrophy (RVH). This patient has severe mitral stenosis with pulmonary congestion, pulmonary hypertension, and right heart failure. A qR in lead V1 is one of the most specific signs for right ventricular hypertrophy. Other causes of a qR in V1 include a right bundle branch block (RBBB) in patients with an anterior myocardial infarction or normal patients who have a QS in V1 and subsequently develop a RBBB. This patient has right axis deviation in addition to the qR in V1 which suggests RVH. Given atrial fibrillation and RVH one should strongly consider mitral stenosis. Additional note on differentiating right ventricular hypertrophy (RVH) from lateral MI: In some cases of severe (RVH) with marked right axis deviation, leads I and aVL may show rS or even QS waves, simulating lateral MI. In this case, lead V1 notably shows a narrow qR complex (not just a tall R wave) strongly suggestive of RVH, rather than posterior-lateral MI. Also, the lateral precordial leads here show rS type complexes, not QR waves. Physical examination, echocardiogram and other imaging modalities also provide important information about RVH vs. lateral MI in cases where the ECG is ambiguous.
  • 8.  
  • 9. The tracing shows sinus rhythm with complete heart block and an A-V junctional type escape rhythm. The P-P interval surrounding an individual QRS complex is narrower (shorter) than the P-P interval between two QRS complexes. Sinus rate variation with complete heart block is called ventriculophasic sinus arrhythmia. The patient has congenital complete heart block. This entity is usually idiopathic but may be secondary to placental transfer of anti-Ro and La antibodies from the mother. The spectrum of disease ranges from patients who are born with severe bradycardia and require pacemakers in infancy to patients who live full life spans without artificial pacemakers. The overall infant mortality is 15%. The escape rhythm in this instance is proximal in the conduction system and a pacemaker is not required at this time.
  • 10.  
  • 11. Ventricular paced rhythm with underlying complete heart block (P waves march through). Most important is superimposed current of injury with hyperacute ST-T changes inferiorly and laterally with reciprocal change V1-V3 consistent with acute infero-postero-lateral ischemia/ myocardial infarction (MI). Pacemaker patterns, like left bundle branch block (LBBB), often mask acute or chronic MI. But sometimes, as in the present case, the ischemic changes &quot;show through.&quot;
  • 12.  
  • 13. Acute infero-lateral and probably posterior myocardial infarction (MI) with inferior Q waves and hyperacute ST-T complexes inferiorly and laterally with reciprocal ST depressions V1-V3. Tall initial R waves in V1-V3 with right bundle branch block. There is second degree AV block (Type 1) with 2:1 block initially and then 3:2 AV Wenckebach with underlying sinus rate of about 85/min. Left axis deviation is present. However, since this is an inferior MI, a pacemaker is not usually indicated: block here is in the AV node usually due to ischemia and increased vagal tone. In contrast, new right bundle with left axis with acute anterior-septal MI would be indication for prophylactic pacemaker. AV block in that setting is due to Type II mechanism associated with severe involvement of His-Purkinje system and carries ominous prognosis with high risk of complete heart block with slow (or no) escape rhythm.
  • 14. We used to see this more often, back in the day…
  • 15. Be careful here. This may look superficially like sinus but if you look in V1, you can see P waves just before the QRS (there is right bundle branch block (RBBB) and also P waves in the T wave (halfway in between) which are partly hidden. Atrial rate is about 150, ventricular rate 75. This is atrial tachycardia with 2:1 block (paroxysmal atrial tachycardia) &quot;PAT with block&quot; if it occurs paroxysmally) which may be due to digoxin toxicity, atrial disease, etc. If you got this right, PAT yourself on back; if you choked on it, a more forceful interscapular blow may be indicated.
  • 16.  
  • 17. The major finding is narrow complex tachycardia with atrial rate of about 300 indicating atrial flutter and slightly variable ventricular response (2:1 conduction, some 3:1 and probably some Wenckebach). Atrial flutter is sometimes mistaken (by housestaff at other institutions) for sinus tachycardia. Look, for instance, at V1 where the flutter waves are hard to see vs. lead II where they are more apparent. If only a V1 type lead were available, the mechanism could be readily mistaken.
  • 18.  
  • 19. Sinus tachycardia with 2:1 block (hidden P waves in T waves, e.g. leads V2 and V3) and left bundle branch block (LBBB). This most likely is type 2 block given its occurrence with sinus tachycardia and presence of LBBB. Patient was referred for evaluation for pacemaker. Remember the &quot;meet me halfway&quot; rule: always look halfway in between obvious P waves for hidden ones.
  • 20. This one is tricky. Be careful.
  • 21. Note there is evidence here of mitral valvular disease: ECG is highly suggestive of biventricular hypertrophy with left ventricular hypertrophy (LVH) type pattern in precordial leads and QRS axis of about +90 degrees. This combination is common with Rheumatic Heart Disease with MS/MR +/- aortic disease. Rhythm is junctional with retrograde P waves with relatively long RP interval in lead II, V1, etc. with single ventricular premature beat. But don’t stop there: of note is long Q-T with prominent U waves raising question of drug effect/toxicity or hypokalemia. Patient was on a phenothiazine here. The ST-T changes are consistent with hypertrophy, drug effect, ischemia, etc. Patient was also on digoxin with low therapeutic level.
  • 22.  
  • 23. Atrial flutter. Don't miss hidden atrial (F) wave just after QRS. This is atrial flutter with 2:1 conduction. Note that with atrial flutter there aren't P waves per se but atrial waves. So if you talk about atrial flutter do not say &quot;the P waves....&quot; since once you say P wave, the informed listener will probably assume it's not atrial fibrillation or atrial flutter. On the other hand, depending on whom you are talking to, this may not be an issue. Also note left ventricular hypertrophy (LVH) and left axis with possible prior inferior wall myocardial infarction (MI) here. Note that typical flutter waves have negative polarity in lead II, as is the case here.
  • 24.  
  • 25. This is highly suggestive of Parkinson's disease. The ECG shows the classic pseudo atrial flutter/fib type waves associated with Parkinsonian tremor. The marked amplitude of the waves, absence of an irregular ventricular rate and subtle hidden true P waves in selected leads (lead 2, intermittently, aVR) indicate the diagnosis. Refer to neurology not cardiology.
  • 26.  
  • 27. This is sinus rhythmn. This is a case of pseudo-flutter waves due to artifact (probably Parkinsonian tremor). Some leads show clear sinus P wave, e.g. V2, V3. So, remember, look at all the leads.
  • 28.  
  • 29. AV nodal reentrant tachycardia (AVNRT). This tracing demonstrates a REGULAR narrow complex tachycardia at a rate of 150. The differential diagnosis of this supraventricular tachycardia includes: sinus tachycardia, AVNRT, automatic atrial tachycardia, atrio-ventricular reentrant tachycardia (AVRT), involving retrograde conduction over a &quot;concealed&quot; bypass tract, or atrial flutter with 2:1 block. If P waves can be located it can be helpful in determining the mechanism of the tachycardia. In this tracing P waves can be located at the end of the QRS in lead II and aVF, producing a &quot;pseudo S&quot; wave. Absence of P waves or a &quot;pseudo S&quot; pattern in II, III, or aVF, or a &quot;pseudo R prime&quot; pattern in V1 is characteristic of AVNRT due to near simultaneous activation of the atrium and the ventricle from the AV node. If the diagnosis is still in doubt, adenosine may be useful in slowing conduction in the AV node. Reentrant rhythms (AVNRT or AVRT) may &quot;break&quot; abruptly, converting to sinus; the ventricular response of other atrial tachcyardia or atrial flutter will slow often revealing underlying atrial activity. (Note: the final wide complex waveform is either an artifact or a ventricular premature beat.)
  • 30. A Pause, for understanding...
  • 31. Here is another AVNRT, with those pseudo-S waves:
  • 32. But AVNRT gets confused with the less common AVRT:
  • 33. AVRT is the consequence of the WPW-type congenital bypass tracks
  • 34. The short PR and Delta Wave of WPW
  • 37. Pre-admission testing showed pre-excitation, with subtle (very, very subtle if you missed it) Wolff-Parkinson-White (WPW) pattern. Note shortish PR and delta wave, best seen in V5 as slurring of initial QRS, with tall R (positive delta wave) in V1, negative in aVL, compatable with lateral bypass tract. What is differential diagnosis of tall R in V1? You should consider 1) normal variant, esp children, but also some adults, or lead placement; 2) funny chest/mediastinal configuration with heart more rightward; 3) posterior myocardial infarction (MI) (inferior/lateral ischemia/MI usually also seen); 4) right ventricular hypertrophy (usually right axis); 5) WPW with posterior or lateral bypass tract; 6) right ventricular conduction abnormality; 7) hypertrophic cardiomyopathy (HCM) with or without obstruction; 8) Duchenne muscular dystrophy (due to postero-basal fibrosis).
  • 38.  
  • 39. Atrial fibrillation with the Wolff-Parkinson-White (WPW) syndrome, with conduction down the bypass tract. This is for the most part a wide complex tachycardia with a rate of about 230 beats/min. The differential diagnosis includes 1) ventricular tachycardia, 2) supraventricular tachycardia with aberrancy, and 3) WPW with conduction down the bypass tract. The major clues include the &quot;irregularly irregular&quot; rhythm and the extremely rapid rate. Ventricular tachycardia may be mildy irregular but this degree of irregularity would be unusual at this very fast rate. The short refractory period of certain bypass tracts can allow extremely rapid heart rates, especially during atrial fibrillation. A correct diagnosis is very important because drugs that slow AV conduction (verapamil, beta blockers, digoxin, adenosine) are contraindicated. These drugs can facilitate preferential conduction down the bypass tract and the atrial fibrillation can degenerate to ventricular fibrillation. A drug of choice is IV procainamide and if this is unsuccessful, DC cardioversion should be performed promptly.
  • 40.  
  • 41. Ectopic atrial (non-sinus) tachycardia with 2:1 block and 3:2 AV Wenckebach (beats 4 and 5). Note the negative P waves in lead II indicating a non-sinus pacemaker. Other important abnormalities: right bundle branch block, left anterior fascicular block and a markedly prolonged QT(U) interval. This patient was on digoxin and quinidine. The evidence for quinidine toxicity (assuming the serum potassium is normal and no other factors prolonging ventricular repolarization are identifiable) is the very long QT(U) interval best seen in V2-V5. This finding predisposes to torsade de pointes and may occur with &quot;therapeutic&quot; or &quot;subtherapeutic&quot; levels of quinidine or other class 1A antiarrhythmic drugs. Atrial tachycardia with Mobitz I block also strongly raises the question of digitalis toxicity. With pure 2:1 AV block, it may be impossible from the surface ECG to tell if the non-conducted beats are due to nodal or infranodal block. The clue here centers on beats 4 and 5 where 3:2 Wenckebach is evident, indicating block in the AV node.
  • 42.  
  • 43. It's only NSR if that stands for Not Sinus Rhythm. Note that there are two P's for every QRS (see V1) and that atrial activity at 176/min is negative in lead 2. This is atrial (not sinus) tachycardia with 2:1 block (conduction). Block is almost certainly at AV node level (Type 1 mechanism). Always exclude digitalis excess with this arrhythmia (not present here). ECG also shows low limb lead voltage, left ventricular hypertrophy (LVH) by precordial voltage and slow R wave progression
  • 44.  
  • 45. There is a wide-complex tachycardia with right bundle branch and left axis deviation (RBBB/LAD) pattern (old per hx). The rhythm is NOT sinus but atrial flutter with subtle flutter waves (aVR, II, V1). Note the negative polarity of atrial waves in lead II, characteristic of typical atrial flutter. Atrial flutter with 2:1 conduction is among most commonly missed major rhythms (often called sinus tachycardia by housestaff-- at other institutions, of course). Sustained atrial flutter is likely the major contributor, if not the precipitant of congestive heart failure syndrome here in this elderly man. Note that patients with atrial flutter may be candidates for ablation treatment to interrupt reentrant wave that underlies this rhythm.
  • 46.  
  • 47. Sinus rhythm with AV Wenckebach (Type 1 AV block) is present with 3:2 and 2:1 conduction. Notice progressive PR prolongation in 3:2 cycles with non-conducted sinus P wave followed by normal PR interval. Etiologies include drugs (beta-blockers, digitalis, calcium channel blockers), inferior ischemia, intrinsic AV node disease, Lyme disease, etc. In healthy young adults with high vagal tone (e.g. athletes) AV Wenckebach during sleeping hours is not uncommon. This patient's ECG also shows left ventricular hypertrophy (LVH), left atrial abnormality (LAA) (biphasic P wave with broad negative component in V1) and infero-lateral T wave inversions raising question of ischemia.
  • 48.  
  • 49. Atrial flutter with 2:1 conduction. Don't overlook subtle &quot;extra&quot; atrial wave (it's NOT a true P wave, but a flutter wave) in the early ST segment. Atrial flutter with 2:1 conduction (block) is often mistaken for sinus tachycardia or paroxysmal supraventricular tachycardia (PSVT)
  • 50.  
  • 51. There is a wide-complex tachycardia. The QRS shows a classic left bundle branch block (LBBB) pattern. If you look carefully, you will see atrial activity in the limb leads, with negative polarity in lead II, at rate of 320/min. Hence, atrial flutter with 2:1 conduction and LBBB.
  • 52.  
  • 53. It may look like sinus tachycardia, but look again in V1: There are two atrial waves for each QRS, so the atrial rate is 260. This is another case of atrial flutter with 2:1 conduction, one of the most commonly mistaken arrhythmias! This case is particularly tough because of low voltage in limb leads.
  • 54.  
  • 55. ECG shows sinus rhythm at 75/min but with 2:1 AV block. There is also left atrial abnormality (LAA). Conducted QRS complexes show right bundle branch block (RBBB) with left axis deviation, most c/w left anterior fascicular block. Minuscule R waves in II, III, aVF also raise question of prior inferior myocardial infarction (MI). The normal PR interval in conducted beats and the RBBB favor an infranodal location of block. Aficionados may also note the ventriculophasic sinus arrhythmia (P-P interval with QRS in-between is shorter than P-P without intervening QRS due to vagal tone fluctuations). Treatment: pacemaker.
  • 56.  
  • 57. Narrow complex tachyardia at 180/min with probable subtle retrograde P waves just after the QRS (may need to print this out to see in lead aVR and II). Therefore, likely diagnosis is AV nodal reentrant tachycardia (AVNRT); although, concealed bypass tract tachycardia or atrial tachycardia not excluded. This is, of course, NOT sinus and is also NOT atrial flutter (no flutter waves and ventricular rate is too fast for flutter with 2:1 conduction).
  • 58.  
  • 59. The underlying rhythm is atrial fibrillation (AF) with a slow regularized ventricular response (45/min) consistent with complete heart block (CHB) with a junctional escape mechanism. The QRS complexes show an intraventricular conduction delay (IVCD) with left axis and left ventricular hypertrophy (LVH) and there is Q-T(U) prolongation. Etiologic differential diagnosis includes intrinsic conduction disease, rule out drug toxicity or metabolic factors. The long QT here is against digitalis as sole factor (digitalis shortens the QT). Hypokalemia or relevant drug effects, such as sotalol or amiodarone, should be excluded. Treatment: If no reversible causes, patient will likely require an implanted ventricular pacemaker, as well as anticoagulation for AF.
  • 60.  
  • 61. Atrial tachycardia with 2:1 block and ventricular premature complexes (VPBs). Check out lead V1 if you missed this. The atrial rate is 200. (Points against atrial flutter are the atrial rate and discrete-appearing P waves, upright in lead 2). Also present: left ventricular hypertrophy (LVH) with intraventricular conduction defect (IVCD) (incomplete left bundle branch block-LBBB) and poor R wave progression (cannot rule out prior anterior myocardial infarction (MI); but this could be due to LVH/IVCD alone); also non-specific ST-T changes. The patient was not digitalis toxic.
  • 62. Major Segue! Homework Assignment for the interns: Ten signs or symptoms with names derived from mythology.
  • 64. This does indeed look like atrial flutter or coarse atrial fibrillation with regularized response. However, if you look closely, (V4; also lead 2 rhythm strip at end) you can see true P waves buried in baseline artifact. So, probable tremor artifact.
  • 65.  
  • 66. Atrial flutter with 2:1 block. Note that there are 2 atrial waves for each QRS in lead V1. The atrial rate is about 240; therefore, this could easily be misread as sinus tachycardia or paroxysmal supraventricular tachycardia (PSVT) variant--not by you of course. ECG also shows left ventricular hypertrophy (LVH) and possible prior antero-septal myocardial infarction (MI), along with non-specific ST-T changes.
  • 67.  
  • 68. Atrial fibrillation with an extremely rapid ventricular response (overall average about 204 beats per minute). There is intermittent right bundle branch block (RBBB) aberrancy (note QRS morphology in V1/V6, especially), associated with the very fast rates. In all, there are 10 consecutive beats with RBBB aberrancy, during which the ventricular response rate is up to 240 beats per minute
  • 69.  
  • 70. This looks like sinus bradycardia at a rate of 43 bpm, BUT... if you look more closely, there are P waves partly hidden in T waves. So this is actually pure sinus with blocked atrial premature complexes (APBs) in a bigeminal pattern. As a result the effective sinus rate is very slow. There are also non-specific T wave inversions noted in V1-V3.
  • 71.  
  • 72. The rhythm is sinus with long, atypical AV Wenckebach cycles (e.g., probable 6:5 block). The reason for the type 1 second degree AV block is apparent from the rest of the tracing which shows classic findings of an acute inferior/right ventricular myocardial infarction, consistent with proximal to mid-occlusion of the right coronary artery (RCA). The finding of ST elevation in lead III>II, as present here, is also consistent with this finding.
  • 73.  
  • 74. The rhythm is sinus with ventricular bigeminy and late diastolic premature ventricular complexes (PVCs) that come just after the second sinus P waves. The PVCs have a right bundle branch block (RBBB) morphology, but this is not intermittent RBBB or Wolf-Parkinson-White Syndrome (WPW).
  • 75.  
  • 76. The rhythm is sinus (or possibly atrial) tachycardia with a P wave rate of about 120/min with AV Wenckebach. There is an evolving inferior (posterior) myocardial infarction (MI) pattern with Q waves and T wave inversions in 2, 3, aVF, and a tall R in V2. The AV conduction disturbance is likely related to the MI. There is also low voltage and non-specific lateral ST-T changes.
  • 77.  
  • 78. Normal sinus rhythm with artifact in V4-V6, simulating premature ventricular complexes (PVCs). Note that simultaneous lead II (bottom) does not show this, and normal rhythm is not affected by the spurious &quot;beats.&quot;
  • 79.  
  • 80. Classic Wolff-Parkinson-White (WPW) pre-excitation pattern, with triad of short PR, wide QRS and Delta waves. Note the polarity of the delta waves (most positive in lead 2 and lateral chest leads) consistent with a right-sided bypass tract. Patients with WPW may not only have atrio-ventricular reentrant tachycardia involving the bypass tract, but also AV nodal reentrant tachycardia, as well as atrial fibrillation (AF), etc.
  • 81. One other segue -- here is WPW in reverse: That is, White-Parkinson-Wolff
  • 82.  
  • 83. Sinus with 5:4 AV Wenckebach. Low voltage (consistent with obesity, Chronic Obstructive Pulmonary Disease (COPD), and/or myocardial infarction (MI) with extensive cardiac muscle necrosis) makes identification of the very broad, low amplitude P waves difficult! Note subtle changes in T-wave morphology due to superimposed atrial activity. (Right axis deviation here could be due to COPD, lateral extension of infarct, or left posterior fascicular block.)
  • 84.  
  • 85. Normal sinus rhythm with a marked prolonged PR (&quot;1st degree AV block&quot;). The P waves, partly obscured by the T waves of the preceding sinus beats, are more clearly evident after the premautre ventricular complexes (PVCs), (beats 3 and 9, and the last beat). In addition, there is an intraventricular conduction delay (IVCD), borderline left axis deviation, and lateral ST-T wave changes associated with a history of multiple non-Q wave myocardial infarctions. The QT appears long in part because of the superimposed P wave deflection at the terminal portion of the T wave.
  • 86.  
  • 87. Sinus rhythm with atrial bigeminy (this pattern starts after the 4th sinus beat). In addition, there are voltage criteria for left ventricular hypertrophy (LVH), possible left atrial abnormality, and poor R-wave progression in V1-V3. The latter finding could be due to LVH, underlying anterior myocardial infarction (MI), lead placement; or could be a physiologic variant. The seventh QRS complex exhibits aberrancy, with a left anterior hemiblock morphology.
  • 88.  
  • 89. This is most likely atrioventricular reentrant tachycardia (AVRT), one of the &quot;short R-P&quot; tachycardias. Note the inverted P waves in leads II, III, and F; with upright P waves in aVR, consistent with retrograde activation of the atria from the region near the AV node and septum. Therefore, the descending limb of this reentrant rhythm is down the AV node, and the ascending limb is up a bypass tract located close to the septum. The differential diagnosis also includes AV nodal reentrant tachycardia (AVNRT) with a somewhat prolonged retrograde (fast pathway) limb, or, less likely, atrial tachycardia coming from low in the atria, with a very prolonged PR interval.
  • 90.  
  • 91. At first glance this might appear to be &quot;sinus tachycardia.&quot; However, a second look reveals two interesting findings. Note the unusual P wave axis (negative in I and aVL) and the short PR interval (or very long RP interval). One possible explanation for these findings is that this is an atrial tachycardia with either 1) a rapidly conducting (&quot;slick&quot;) AV node, or 2) an &quot;atrio-His&quot; bypass tract with fast conducting properties. This would account for the short PR interval in the presence of an atrial tachycardia. Another possibility for this non-sinus tachycardia is that it represents a &quot;long RP tachycardia&quot; involving a left lateral bypass tract which is slowly conducting (to account for the long RP interval). In this latter case, the reentrant circuit would involve antegrade (orthodromic conduction) down a rapidly conducting AV node and through the ventricular myocardium, with retrograde conduction up the slowly conducting bypass tract, and then across the atria back to the AV node.Note that resting heart rates above 140 beats/min are rarely due to sinus tachycardia in the very elderly.
  • 92.  
  • 93. The rhythm is atrial fibrillation with a rapid ventricular response (irregular rate at about 120 beats/min). There is an extremely wide QRS of left bundle branch block morphology; the QRS duration of over 200ms is suggestive of drug effect or hyperkalemia. Flecainide is a class Ic (Na+ channel blocking) drug, which exhibits &quot;use dependency&quot;: at faster rates with less time for dissociation of the drug from the channel, Na+ channels are increasingly saturated with the drug. This increases the degree of QRS prolongation, and may be mistaken for ventricular tachycardia.
  • 94.  
  • 95. Ventricular tachycardia (VT) at rate of 170. The right bundle branch block morphology is atypical (monomorphic R, rather than rSR', in V1), and the R:S ratio is less than 1 in V6, both suggestive of ventricular tachycardia. Although the most common underlying diagnosis in adult North American patients with sustained monomorphic VT is coronary heart disease status post myocardial infarction (MI)s, this patient had a non-ischemic cardiomyopathy. The morphology of the VT is suggestive of origin from the left side of the heart, near the base (right bundle branch block with inferior/rightward axis). As with any wide complex tachycardia that is hemodynamically tolerated, the most important data to obtain is the 12-lead ECG, as this can direct possible radiofrequency ablation at the time of electrophysiology study.
  • 96. (This is quite difficult, but worth it.)
  • 97. Atrial flutter (negative atrial waves in II, positive in V1) with an atrial rate of approximately 230 BPM with subtle group beating. On close inspection, there are groups of two ventricular beats followed by pauses, with more atrial waves than there should be for simple 3:2 AV Wenckebach conduction pattern. This pattern is attributable to dual levels of block (multilevel block) in the AV node: in this case 2:1 AV conduction &quot;above&quot; 3:2 AV Wenckebach conduction. If you said atrial flutter (or raised question of flutter vs.rapid atrial tachycardia) along with Wenckebach variant, give yourself four out of five stars.
  • 98.  
  • 99. This is most consistent with accelerated idioventricular rhythmn (AIVR), originating from the left ventricle and therefore accounting for the atypical RBBB morphology. ST elevations in the precordial leads are likely due to the injury current from and underlying acute myocardial infarction (MI). AIVR may be marker of reperfusion following acute MI. The rate of about (83 per minute) is too slow for ventricular tachycardia and too fast for complete heart block.
  • 100.  
  • 101. This is relatively slow bi-directional ventricular tachycardia (VT). Note the sinus P waves (sinus tachycardia) at a rate of about 110 beats/min (most visible in the lead II rhythm strip toward end of recording) with apparent atrioventricular (AV) dissociation and a wide complex tachycardia at about 100 beats/min. The QRS complex alternates between two morphologies. This arrhythmia should always raise suspicion of digitalis toxicity (not present in this patient with a recent myocardial infarct). The name, bi-directional VT, is confusing, as are many terms generated by cardiologists, who in the main, are poorly literate in my estimation. It refers to alternating right-and left- BBB conduction of the ectopic ventricular focus. “Ventricular remodeling” is another term that drives me crazy; it evokes images of rearranging the furniture in the LV chamber.
  • 102.  
  • 103. Atrial fibrillation with a very rapid ventricular response (average 160-170/min).Other Findings: Low limb voltage, minor RV conduction delay with rsr' (or possibly Qr) in V1 complex, and non-specific ST-T changes (on digoxin).
  • 104.  
  • 105. Coarse atrial fibrillation (not flutter) is present with a slow ventricular response. There is an atypical left bundle branch block (LBBB) pattern. The rsR' in lateral leads (e.g., V6 here) is highly suggestive of severe left ventricular (LV )dysfunction, often associated with a ventricular aneurysm (El-Sherif sign). A QR or rsR' complex is also present in I and aVL. Left axis deviation and a long QT are noted, as well. The patient had heart failure due to prior silent infarction.
  • 106.  
  • 107. Atrial tachycardia (NOT sinus tachycardia) with variable block. Also left ventricular hypertrophy (LVH) and left axis deviation are present. This is not complete heart block either. Note that the rate is not slow and regular. Instead, the changing PR is due to variable AV block. This patient has a history of rheumatic heart disease (mitral and aortic regurgitation) and underwent a cardiac electrophysiology procedure with ablation of the ectopic atrial site that was the source of the tachycardia. Always exclude digoxin excess (not present here) in cases of atrial tachycardia with block as well. (Note: the subtle variation in QRS morphology, e.g., in leads II,IIII and aVF, which is due to varying degrees of superposition of the P waves on the QRS complexes and also, possibly, to slight changes in QRS conduction at different rates. This nonspecific finding should be distinguished from 2:1 electrical alternans patterns.)
  • 108. Now let’s cover some ventricular issues:
  • 109. Causes of wide-QRS tachycardia
  • 111. Differential criteria considering the QRS morphology in leads V1 and V6 in case of right bundle branch block (RBBB) or left bundle branch block (LBBB) aspect.
  • 112. Our first unknown. Rapid SVT or V. Tach? Makes a difference, right?
  • 113. We’ll start with the Brugada algorithm
  • 114. Are there any RS complexes in the V leads? Absence of any RS complex in the precordial leads V1..V6 is 100% specific for VT, but is insensitive (26%).
  • 115. Here’s the tracing once again. So, is there an RS in any of the precordial leads?
  • 116. Yes, V2 has an RSR’. So let’s go to the next step of the Brugada Algorithm...
  • 118. In any of these RS complexes, is the interval from start of R to nadir of S > 100ms ?
  • 119. Look at V2, and specifically at the fifth complex there... The R wave starts on a solid line, and the nadir of the S wave is only two small boxes or .08 secs (80 msecs) out.
  • 120. So we need to go to the next step in the algorithm...
  • 121. Is there AV dissociation, or are there fusion or capture beats? {A capture beat QRS looks normal because a conducted sinus beat transiently captures the ventricles;
 fusion beat's QRS is intermediate between normal and the VT complex}
  • 122. Well, what do you think? I think I can see P waves. Can you?
  • 123. So, if we are correct, then according to the algorithm this is V. Tach, because there are P waves not associated with any QRS’s. But let’s suppose we aren’t sure. Let’s go to the next step of the B.A., that is examining morphology. (This is tedious, but productive.) Is the QRS complex in V1 predominantly positive?
  • 124. No it isn’t. So this doesn’t help us. Let’s move on...
  • 125. The RS interval is over 60 ms., so...
  • 126. Next look at V6. Is there a monophasic R , or a QR , or an R:S ratio < 1 , or a QS? If any of these morphologic criteria exist, it’s likely to be V tach. What’s your call?
  • 127. This is a rapid monomorphic ventricular tachycardia (VT), 280 beats per minute, associated with hemodynamic collapse. This tracing was obtained from a patient with severe ischemic cardiomyopathy during an electrophysiologic (EP) study. The rhythm later converted to sinus with a single external shock. This patient had an atrial rate of 72 beats per minute (measured with intracardiac electrodes, not shown). Although ventriculoatrial dissociation (faster V rate than A rate) is diagnostic of VT, the surface ECG findings are only present approximately 20% of the time. In this tracing, the ventricular rate is simply too fast for P waves to be observed. VT with cycle lengths from 200-240 ms is often termed ventricular flutter. So by morphology, and because we think We see A-V dissociation as well, we can say:
  • 128. But here’s an important point: if there is hemodynamic collapse, shoot first and ask questions later: Often the best 'drug' is in fact electricity, as sustained VT often deteriorates into VF if left unmanaged.
  • 129. Okay, next case: Is this supraventricular, or nonsustained V. Tach?
  • 130. If we use the algorithm...
  • 131. You will readily see that two sensitive criteria for V. Tach are met: There is A-V dissociation, and in V1, the interval from the start of the R wave to the nadir of the S wave is > 100 msec.
  • 132. Can you see the P waves, without corresponding QRS’s? (A-V dissociation) And in V1, the first complex starts on a solid Line and the nadir of the S wave is at least 2.5 boxes later (i.e. .10 sec. or 100 msec)
  • 133. So this is repetitive monomorphic ventricular tachycardia (VT) from an asymptomatic 45-year-old female wind surfer. This patient has a structurally normal heart. This ECG pattern is typical for a form of idiopathic VT arising from the right ventricular outflow tract. Unlike ischemic VTs, this form of VT is often provoked by exercise and suppressed by beta blockade or verapamil. The prognosis for these patients is good, with the following 2 exceptions: (1) occasionally, this form of VT is associated with right ventricular dysplasia, which is associated with sudden death, and (2) occasionally, patients with incessant VT develop congestive heart failure due to tachycardia-induced cardiomyopathy. The cardiomyopathy generally resolves when the tachycardia is treated.
  • 135. You might be tempted to jump to “atrial fibrillation with an aberrated ventricular conduction” but let’s use the algorithm first.
  • 137. There are RS’s in the precordial leads, so that criteria is not met, those are very broad S waves (not to be confused with inverted broad ST/T segments) so the RS interval is certainly greater than 100 msec.
  • 138. Agree? Also, look at V4-6. Remind you of anything?
  • 139. This ECG shows a relatively regular, wide complex polymorphic ventricular tachycardia, rate 150 bpm, that is most likely a type of torsade(s) de pointes. See leads V4-V6 for classic “turning of the points” morphology. Intermittent ventricular fusion beats may be present. Causes of this life-threatening arrhythmia include a variety of drugs and specific electrolyte abnormalities (especially hypokalemia and hypomagnesemia) associated with long QT (U) intervals, as well as hereditary long QT syndromes due to “channelopathies.”Admission laboratory testing revealed a serum potassium of 2.2 (normal range 3.5-5.3 mEq/L), sodium of 143 (range 135-148 mEq/L), bicarbonate of 16 (normal 21-30mEq/L), phosphate 1.0 (range 2.7-4.5 mg/dL) and magnesium 2.5 (range 1.6-2.6 mg/dL.) Polymorphic V. Tach
  • 140. This one should be easy for you now...
  • 141. Because there are RS waves in the precordial leads, that criteria is not met, but the RS interval is huge. Let’s use that other table of criteria on this one, just for fun...
  • 142. Differential criteria considering the QRS morphology in leads V1 and V6 in case of right bundle branch block (RBBB) or left bundle branch block (LBBB) aspect .
  • 143. Do you agree that this looks RBBB-like?
  • 144. Differential criteria considering the QRS morphology in leads V1 and V6 in case of right bundle branch block (RBBB) or left bundle branch block (LBBB) aspect. So, the tracing has a RBBB like appearance, and V1 has a qRSR’.
  • 145. So this one is monomorphic V. Tach The ECG shows a relatively regular, very wide-complex tachycardia with a heart rate of 130/min and a QRS duration of about 180 ms. The QRS morphology is that of a right bundle branch block (RBBB) with a qRSR′in V1, rS in V6 and a marked rightward axis. Underlying atrial activity is hard to discern with certainty but may be sinus with AV dissociation. These factors indicate ventricular tachycardia which was confirmed at cardiac electrophysiology study. A left ventricular tachycardia intramyocardial focus (associated with this ECG pattern) localized below the anterior papillary muscle was successfully eliminated with radiofrequency ablation.
  • 147. Let’s start with this again...
  • 148. There are RS’s in the precordial leads, so that’s out. The RS interval is not greater than 100 msec. so that’s out too.
  • 149. Is there A-V dissociation? Actually, there is A-V ass ociation; Look at lead III
  • 150. So now we have to go through all those laborious morphologic criteria: Is the QRS complex in V1 predominantly positive?
  • 151. Yes it is. So:
  • 152. V1 is a QR So, next look at V6...
  • 153. Ans.: none of the above, so this is SVT, but...
  • 154. A close look at V5 gives us another way to attack this...
  • 155. So this is not V. Tach. This is Atrial flutter with 2:1 conduction and RBBB This ECG shows subtle atrial flutter with 2:1 block, right bundle branch block and right axis deviation. The atrial rate is about 260 bpm (see upright flutter waves in lead V1) with a ventricular response rate of about 130 bpm. The right axis deviation here, in the absence of clinical evidence of right ventricular hypertrophy or lateral infarction, likely represents left posterior fascicular block.The patient had persistent atrial flutter with variable block or atrial fibrillation at other times.
  • 157. This ECG shows underlying coarse atrial fibrillation (AF) with (right) ventricular demand pacing at a rate about 78 bpm with a relatively wide QRS complex (left bundle branch block morphology) at about 180 milliseconds. Two PVCs are seen (2nd and 6th beat).There are no sinus P waves or flutter waves and no atrial spikes to indicate dual chamber pacing. You don’t need algorithms for this one.
  • 159. Back to the algorithm...
  • 160. My problem is knowing where the baseline is in the precordial leads, and therefore defining R, S, and Q waves. So we’ll need to do the morphology thing again. Or use that other chart...
  • 161. Differential criteria considering the QRS morphology in leads V1 and V6 in case of right bundle branch block (RBBB) or left bundle branch block (LBBB) aspect.
  • 162. The rSR’ in V1 suggests a RBBB
  • 163. ...and the QS in V6 gives it away...
  • 164. This is monomorphic ventricular tachycardia at a rate of about 140 bpm with a very wide (QRS duration about 0.20 second) with a right bundle branch block and superior axis deviation morphology. The previous ECG done on this patient the same day showed an extensive anterolateral ST elevation acute myocardial infarction . There is suggestion of ST elevation here in V5 and V6 which is difficult to interpret with the wide QRS here.
  • 166. This ECG shows sinus rhythm with ventricular bigeminy (every other beat is a ventricular premature beat or complex (VPB or VPC, for short) of the same morphology in each lead.Note also the minimal diffuse ST elevations (with non-specific T wave inversions). These repolarization changes are consistent with sub-clinical perioperative pericarditis, a ECG common finding after coronary artery bypass graft (CABG) surgery.
  • 168. The problem here is that it’s impossible for me to fit this into a BBB category (and use the table), and I can’t really define the baseline well enough to use the Brugada algorithm. So let’s see if we can dissect this out...
  • 169. This ECG shows a very wide-complex (between 160-200 msec) tachycardia at a rate about 180 bpm with a bundle branch (BBB) morphology.
  • 170. The diagnosis of monomorphic ventricular tachycardia is strongly supported by the following criteria: 1) Wide QRS (greater than 140 ms) in the absence of drugs or hyperkalemia (the QRS here is extremely wide); 2) wide monophasic R wave in lead V1 (or RS or QR complex in that lead (or whatever we choose to call it).
  • 171. 3) QR complexes in the inferior leads with extreme axis deviation; 4) rS or QS waves in V6. It’s the rS in V6 that makes me worry about V. tach.
  • 172. A resting rate of 180 bpm excludes sinus tachycardia, especially in this age group. There is no evidence of atrial flutter or atrial tachycardia, and, as described, the QRS morphology here is strongly against aberrant supraventricular conduction. This patient had a complicated cardiac history of coronary artery disease, status post bypass graft surgery and prior inferior infarction. He had recurrent ventricular tachycardia, likely originating the area of the prior inferior infarct. An echocardiogram revealed showed a dilated left atrium and left ventricle, extensive thinning with an akinetic scar of the inferior base and moderately severe mitral regurgitation. Estimated left ventricular ejection fraction was 35%.
  • 173. This is a great tracing. I hope you’re saving these.
  • 174. Let’s see if our algorithm works for this one:
  • 175. There are RS complexes in the precordium, but the RS interval is quite long, much greater than 100 msec, so that criteria is fulfilled for V. Tach
  • 176. Now let’s use the table. Is this more RBBB or LBBB?
  • 177. LBBB type. Here’s the table: The R in V1 is not greater than 30 ms, and V6 has an RS, so neither criteria is fulfilled here for V tach .
  • 178. So we have conflicting algorithms. Is there anything else that helps us here? Think outside the box:
  • 179. The first 4 beats of this ECG show termination of a run of wide-complex tachycardia at rate about 150 bpm. The 5th and the 6th beats supraventricular conduction with underlying atrial fibrillation now apparent. The conducted supraventricular beats suggest underlying inferior myocardial infarction with prominent Q waves in leads II and aVF and a borderline intraventricular conduction delay (QRS=110 msec). Starting with the 7th beat and continuing to the end, the wide-complex ventricular tachycardia resumes. The wide-complex tachycardia (QRS about 160 msec) has left bundle branch block morphology, with an inferior QRS axis.This ECG shows two features strongly favoring ventricular tachycardia over supraventricular tachycardia with left bundle aberrancy, namely: 1) a broad R wave (40 msec or more in duration) in leads V2, and 2) a delayed intrinsicoid deflection great than 70 msec in V1-V2 measured from the initial onset of the QRS to the nadir of the S wave. (The absence of these findings does not exclude ventricular tachycardia, however.)This patient ruled in for a small acute myocardial infarction with an elevated troponin, but normal creatine kinase. Echocardiogram showed a major infero-lateral wall motion abnormality consistent with prior myocardial infarction as well. He had recurrent episodes of ventricular tachycardia and underwent successful ablation of an ectopic focus in the crest of the interventricular septum, under the left coronary cusp, consistent with the monomorphic ventricular tachycardia seen here with a left bundle branch block, inferior QRS axis morphology.
  • 180. New case: what do you think?
  • 181. For the sake of brevity, I will tell you that neither the Brugada algorithm nor the table work for this one. That is to say, both will tell us that this is SVT. Well then, what kind of SVT? It’s not flutter. There are no flutter waves. Nor are there any atrial fibrillatory waves. It’s not WPW; there are no delta waves. There are no reentrant atrial waves seen, although after some of the ventricular complexes, one can see P waves. Sometimes the algorithms mislead us. That is why you need to think for yourself. This is hard stuff, but no one can fault you for saying: “I’m still worried about V. Tach.”
  • 182. This ECG shows sinus rhythm (normal QTc) with runs of non-sustained monomorphic ventricular tachycardia (starting 2nd beat, rate 170 bpm). The ventricular beats have a left bundle branch block pattern and inferior/borderline rightward QRS axis raising consideration of repetitive monomorphic ventricular tachycardia arising from the right ventricular outflow tract (RVOT). Isolated premature ventricular complexes (PVCs) and two ventricular couplets are also present with the same morphology.Atrial tachycardia is excluded as there are no premature P waves prior to the runs. The P waves associated with the wide complex beats appear to be retrograde consistent with ventricular-atrial conduction. No atrial flutter waves are seen. There is no delta wave to suggest Wolff-Parkinson-White pattern.The patient had an idiopathic cardiomyopathy with global mild ventricular hypokinesis. Previous attempts at ablation in the RVOT region had been attempted. At the time of repeat cardiac electrophysiologic study, the arrhythmia was not inducible, despite intensive programmed electrical and pharmacologic stimulation. The diagnosis was monomorphic ventricular tachycardia of unknown mechanism, possibly arising from the superior portion of the interventricular septum.
  • 184. This is a classic ECG obtained during a ventricular fibrillation (VF) cardiac arrest. The findings are typical of ventricular fibrillation with more organized ventricular electrical activity in the second half of the strip consistent with ventricular flutter.There is no evidence of any atrial activity ruling out atrial fibrillation with Wolff-Parkinson-White (WPW ) or atrial flutter with a bundle branch block. The characteristic, systematic variation in QRS axis of torsade is not seen here (although torsade may degenerate terminally into ventricular fibrillation.)
  • 185. This is one for the table:
  • 186. The tracing has a classic RBBB configuration And is triphasic both in V1 and V6
  • 187. There is a classic right bundle branch block (RBBB) morphology (rsR ' in V1) making VT unlikely. No definite atrial activity is seen (P waves or flutter waves). The very regular rate at 150/min excludes AF. Therefore, the rhythm is most consistent with paroxysmal supraventricular tachycardia (PSVT), most likely AV nodal re-entrant tachycardia (AVNRT), although AV re-entrant tachycardia or atrial tachycardia (with hidden P waves) are not excluded.
  • 189. There are fixed coupled premature ventricular compexes (PVCs) in a bigeminal pattern. Note the onset of each VPC comes just after a (non-conducted) sinus P wave, giving the appearance of a short PR.The sinus beats show probable left atrial abnormality, borderline left axis deviation, and a minor right ventricular (RV) conduction delay .
  • 190. Why the wide-complex rhythmn?
  • 191. Both atrial and ventricular pacemaker spikes are seen, for example, in lead II, before each P wave and QRS, respectively. The patient has a dual chamber rate responsive pacemaker (DDDR) pacemaker.
  • 193. Complex ECG showing: 1) Underlying atrial fibrillation; 2) Long Q-T(U); 3) Ventricular pacing with intermittent sensing failure; and 4) Run of torsade de pointes-type of polymorphic ventricular tachycardia (second half of record).
  • 194. Very tough. Be careful.
  • 195. Hint: I showed you this one to illustrate the A-V dissociation criteria of the Brugada algorithm Do you see the A-V dissociation?
  • 196. There is a wide complex tachycardia at about 165/min with an right bundle branch block (RBBB) morphology. Underlying sinus tachycardia at about 136/min and AV dissociation are noted, confirming the diagnosis of ventricular tachycardia (VT). (Note the clear sinus P waves, e.g., just after 5th QRS.) The QRS morphology and axis are consistent with origin of the VT from the posterior septum. The patient had had a prior large inferior-posterior myocardial infarction (MI). (Of additional note here is the relatively low limb lead voltage and sinus tachycardia).
  • 198. What helps me here is V6. I get an idea of where the baseline is, and the RS interval here clearly exceeds 100 msecs.
  • 199. Ventricular tachycardia (VT) at rate of 170. The right bundle branch block morphology is atypical (monomorphic R, rather than rSR', in V1), and the R:S ratio is less than 1 in V6, both suggestive of ventricular tachycardia. Although the most common underlying diagnosis in adult North American patients with sustained monomorphic VT is coronary heart disease status post myocardial infarction (MI)s, this patient had a non-ischemic cardiomyopathy. The morphology of the VT is suggestive of origin from the left side of the heart, near the base (right bundle branch block with inferior/rightward axis). As with any wide complex tachycardia that is hemodynamically tolerated, the most important data to obtain is the 12-lead ECG, as this can direct possible radiofrequency ablation at the time of electrophysiology study.
  • 200. Is this ventricular tachycardia?
  • 201. No. The rhythm is atrial fibrillation with a rapid ventricular response (irregular rate at about 120 beats/min). There is an extremely wide QRS of left bundle branch block morphology; the QRS duration of over 200ms is suggestive of drug effect or hyperkalemia. Flecainide is a class Ic (Na+ channel blocking) drug, which exhibits &quot;use dependency&quot;: at faster rates with less time for dissociation of the drug from the channel, Na+ channels are increasingly saturated with the drug. This increases the degree of QRS prolongation, and may be mistaken for ventricular tachycardia. The take-home message here, for this impossible case, is to ask what meds the patient is on. Even if I were told “flecainide for A. Fib” I would not send this patient home.
  • 203. No. This is atrial fibrillation with an extremely rapid ventricular response (overall average about 204 beats per minute). There is intermittent right bundle branch block (RBBB) aberrancy (note QRS morphology in V1/V6, especially), associated with the very fast rates. In all, there are 10 consecutive beats with RBBB aberrancy, during which the ventricular response rate is up to 240 beats per minute (Both the algorithm and the table work well on this one.)
  • 205. No. This is a ventricular paced rhythm with underlying complete heart block (P waves march through). Most important is superimposed current of injury with hyperacute ST-T changes inferiorly and laterally with reciprocal change V1-V3 consistent with acute infero-postero-lateral ischemia/ myocardial infarction (MI). Pacemaker patterns, like left bundle branch block (LBBB), often mask acute or chronic MI. But sometimes, as in the present case, the ischemic changes &quot;show through.&quot;
  • 206. Think this one through with me. Forget about algorithms.
  • 207. Atrial fibrillation with the Wolff-Parkinson-White (WPW) syndrome, with conduction down the bypass tract. This is for the most part a wide complex tachycardia with a rate of about 230 beats/min. The differential diagnosis includes 1) ventricular tachycardia, 2) supraventricular tachycardia with aberrancy, and 3) WPW with conduction down the bypass tract. The major clues include the &quot;irregularly irregular&quot; rhythm and the extremely rapid rate. Ventricular tachycardia may be mildy irregular but this degree of irregularity would be unusual at this very fast rate. The short refractory period of certain bypass tracts can allow extremely rapid heart rates, especially during atrial fibrillation. A correct diagnosis is very important because drugs that slow AV conduction (verapamil, beta blockers, digoxin, adenosine) are contraindicated. These drugs can facilitate preferential conduction down the bypass tract and the atrial fibrillation can degenerate to ventricular fibrillation. Call EP fellows stat! A drug of choice is IV procainamide and if this is unsuccessful, DC cardioversion should be performed promptly.
  • 208. Summary: Many of these are very difficult. The algorithms are hard to commit to memory. For family medicine practitioners who see this stuff infrequently, this can be an impossible situation, unless you remember this axiom:
  • 209. Always assume the patient has the worst treatable condition you can do something about, and proceed from there. In the case of wide-complex tachycardias, to say: “This is probably just SVT with aberrated conduction” and send the patient home, is to court disaster.
  • 210. You do not have to know everything to be a great doctor. You just have to think.